82
JULY 2016 PENNSYLVANIA BAR EXAMINATION Essay Questions and Examiners’ Analyses and Performance Test Pennsylvania Board of Law Examiners 601 Commonwealth Avenue, Suite 3600 P.O. Box 62535 Harrisburg, PA 17106-2535 (717) 231-3350 www.pabarexam.org ©2016 Pennsylvania Board of Law Examiners

JULY 2016 PENNSYLVANIA BAR EXAMINATION · JULY 2016 PENNSYLVANIA BAR EXAMINATION Essay Questions and Examiners’ Analyses and Performance Test Pennsylvania Board of Law Examiners

Embed Size (px)

Citation preview

Page 1: JULY 2016 PENNSYLVANIA BAR EXAMINATION · JULY 2016 PENNSYLVANIA BAR EXAMINATION Essay Questions and Examiners’ Analyses and Performance Test Pennsylvania Board of Law Examiners

JULY 2016

PENNSYLVANIA BAR

EXAMINATION

Essay Questions and Examiners’ Analyses

and

Performance Test

Pennsylvania Board of Law Examiners

601 Commonwealth Avenue, Suite 3600

P.O. Box 62535

Harrisburg, PA 17106-2535

(717) 231-3350

www.pabarexam.org

©2016 Pennsylvania Board of Law Examiners

Page 2: JULY 2016 PENNSYLVANIA BAR EXAMINATION · JULY 2016 PENNSYLVANIA BAR EXAMINATION Essay Questions and Examiners’ Analyses and Performance Test Pennsylvania Board of Law Examiners

i

Table of Contents

Index ..................................................................................................................................................ii

Question No. 1: Facts and Interrogatories, Examiner's Analysis and Grading Guidelines ..............1

Question No. 2: Facts and Interrogatories, Examiner's Analysis and Grading Guidelines ..............9

Question No. 3: Facts and Interrogatories, Examiner's Analysis and Grading Guidelines ..............18

Question No. 4: Facts and Interrogatories, Examiner's Analysis and Grading Guidelines ..............25

Question No. 5: Facts and Interrogatories, Examiner's Analysis and Grading Guidelines ..............34

Question No. 6: Facts and Interrogatories, Examiner's Analysis and Grading Guidelines ..............43

Performance Test and Grading Guidelines ........................................................................................52

Page 3: JULY 2016 PENNSYLVANIA BAR EXAMINATION · JULY 2016 PENNSYLVANIA BAR EXAMINATION Essay Questions and Examiners’ Analyses and Performance Test Pennsylvania Board of Law Examiners

ii

Index

Question No. 1

1. Decendents’ Estates: effect of pending divorce on will

2. Decendents’ Estates: anti-lapse statute

3. Federal Income Tax: barter income and business/personal expense

4. Professional Responsibility: conflict of interest

Question No. 2

1. Torts: strict product liability

2. Torts: damages

3. Evidence: expert testimony

4. Civil Procedure: default judgment

Question No. 3

1. Criminal law: murder

2. Criminal law: search

3. Evidence: privilege

4. Family law: prenuptial agreements

Question No. 4

1. Constitutional law: First Amendment

2. Employment law: Title VII race discrimination

3. Civil Procedure: jurisdiction

Page 4: JULY 2016 PENNSYLVANIA BAR EXAMINATION · JULY 2016 PENNSYLVANIA BAR EXAMINATION Essay Questions and Examiners’ Analyses and Performance Test Pennsylvania Board of Law Examiners

iii

Question No. 5

1. Property: joint tenancy with right of survivorship

2. Property: transfer of title

3. Contracts: unilateral contract

4. Contracts: parol evidence

Question No. 6

1. Professional Responsibility: corporate client

2. Corporations: shareholder derivative suit

3. U.C.C. Art. II: additional term in confirming memorandum

4. U.C.C. Art II: express warranty

Page 5: JULY 2016 PENNSYLVANIA BAR EXAMINATION · JULY 2016 PENNSYLVANIA BAR EXAMINATION Essay Questions and Examiners’ Analyses and Performance Test Pennsylvania Board of Law Examiners

Question No. 1

Roger lived in and operated a successful office cleaning service as a sole proprietorship

in E County, Pennsylvania. He established the cleaning service before he married Peg in 2011,

and the cleaning service remained in his sole ownership and control after the marriage. Roger

and Peg had no children together, but Roger had two adult children, Michael and Wanda, from

his previous marriage.

In June of 2013, Roger contacted Larry, an E County attorney, to make a will for him.

Larry drafted the will on Roger’s instruction several days after the contact, and it was properly

signed and witnessed, with the essential terms as follows:

I hereby devise and bequeath all of my estate, real and personal, in equal one-third

shares to my wife Peg, and to each of my children, Michael and Wanda. I appoint

Peg as Executrix.

In early February of 2015, Roger and Peg argued about numerous issues. Peg abruptly

left the marital home and told Roger she wanted a divorce, to which Roger agreed. Peg confided

in her boss, Sam, that she needed a good divorce lawyer but knew they were expensive. Sam

told Peg he knew of an excellent lawyer, Angela, who did inexpensive divorce work. Before

Peg’s appointment with Angela, Sam telephoned Angela to ask her to quote Peg a reduced

hourly rate and stated that he would pay any additional amount Angela would ordinarily charge.

Sam also instructed Angela not to tell Peg about their arrangement. Angela complied with this

request and represented Peg, filing a no-fault divorce complaint on the basis that the marriage

was “irretrievably broken” along with a request for equitable distribution on March 1, 2015.

Angela properly served the complaint on Roger the next day. Larry represented Roger.

The lawyers proposed property settlement agreements, but the parties did not agree. Sam

paid Angela substantial money for Peg’s legal fees and again told Angela not to inform Peg.

1

Page 6: JULY 2016 PENNSYLVANIA BAR EXAMINATION · JULY 2016 PENNSYLVANIA BAR EXAMINATION Essay Questions and Examiners’ Analyses and Performance Test Pennsylvania Board of Law Examiners

Angela complied. In July 2015, Roger and Peg agreed to “bifurcate” the proceedings to obtain a

no-fault divorce decree and to resolve the marital property issue later in court, as is permitted by

the Divorce Code. Both Peg and Roger promptly and properly filed affidavits with the

prothonotary of the court consenting to divorce.

Two days after the consents were filed, Roger’s son, Michael, was killed in a work

accident; Michael was survived by his wife and two minor children. Roger was grief-stricken

and on July 30, 2015, died of a sudden heart attack. The divorce decree had not been entered.

Roger had made an arrangement with Larry a few years earlier in which Roger would

provide cleaning services to Larry’s law office suite in exchange for legal services Larry

provided to Roger for personal or business matters. Larry had previously paid another cleaning

provider as an expense of his law practice. In calendar year 2015, Larry had done legal work

worth $2,000 for Roger solely for his divorce that year, and Roger provided cleaning services at

Larry’s law office that were also worth $2,000 for 2015. Shortly before Roger’s death, Larry

and Roger agreed that this was a fair exchange of the value of their services at that time. Both

Roger and Larry report business as well as personal income as individuals.

1. Peg properly filed Roger’s will with the Register of Wills. Wanda filed a contest

to the will, challenging the devise to Peg of a 1/3 estate share and Peg’s

appointment as executrix because of the ongoing divorce proceedings. How

would the court likely rule?

2. Assuming, solely for this issue, Roger and Peg had not separated and that there

was no divorce action filed or pending, how would Roger’s estate be distributed

under his will in light of Michael having pre-deceased Roger?

3. Both Larry and Roger are cash basis and calendar year tax filers. How should the

exchange of legal services and office-cleaning work by Roger and Larry in 2015

be treated for federal income tax purposes by Roger’s estate and Larry?

4. Was Angela in violation of any of the Rules of Professional Conduct by accepting

Sam’s supplemental payments for Peg’s divorce?

2

Page 7: JULY 2016 PENNSYLVANIA BAR EXAMINATION · JULY 2016 PENNSYLVANIA BAR EXAMINATION Essay Questions and Examiners’ Analyses and Performance Test Pennsylvania Board of Law Examiners

Question No. 1: Examiner’s Analysis

1. The court would likely rule that Peg’s pending divorce from Roger nullified both

her 1/3 share of his estate and her appointment as executrix because grounds for

divorce were established when both parties filed their affidavits of consent to

divorce with the court.

The issue is the effect of the pending divorce of Peg and Roger on the provisions of

Roger’s will devising 1/3 of his estate to Peg and her appointment as executrix under the

Pennsylvania Probate, Estates and Fiduciaries Code (PEF Code).

The PEF Code provides the following regarding the effect of a pending divorce on the

terms of a will:

Any provision in a testator’s will in favor of or relating to the testator’s spouse

shall become ineffective for all purposes unless it appears from the will that the

provision was intended to survive a divorce, if the testator:

* * *

(ii) dies domiciled in this Commonwealth during the course of divorce

proceedings, no decree of divorce has been entered pursuant to 23 Pa. C.S.A. §

3323 (relating to decree of court) and grounds have been established as provided

in 23 Pa. C.S. § 3323(g).

20 Pa. C.S.A. § 2507(2). This invalidation provision undoubtedly includes the right or

designation to serve as the executrix of the will of a divorced spouse, as the statute states that

“any provision . . . in favor of or relating to the testator’s spouse shall become ineffective for all

purposes . . . .” Id.; See Bloom v. Selfon, 520 Pa. 519, 522, 555 A.2d 75, 77 (1989) (stating

“upon divorce, testamentary provisions favoring one’s spouse” are ineffective and finding

provision in testator’s will appointing husband as executor was rendered inoperative by the

divorce of the parties). There is nothing in the facts to establish that Roger’s will gave any

indication that Peg’s bequest of 1/3 of his individual estate, or the appointment of Peg as

executrix should survive a divorce.

The establishment of grounds for divorce is defined by the Divorce Code at 23 Pa. C.S.A.

§ 3323(g), which provides, in pertinent part, as follows:

(g) Grounds established – For purposes of subsections (c.1) [bifurcation] and (d.1)

[death of a party], grounds are established as follows:

* * *

(2) In the case of an action for divorce under section 3301(c), both parties

have filed affidavits of consent.

3

Page 8: JULY 2016 PENNSYLVANIA BAR EXAMINATION · JULY 2016 PENNSYLVANIA BAR EXAMINATION Essay Questions and Examiners’ Analyses and Performance Test Pennsylvania Board of Law Examiners

The facts set forth that Roger lived in and operated a business in E County, Pennsylvania

and therefore was domiciled in Pennsylvania at the time of his death. They also provide that Peg

had left Roger and the marital residence in February 2015 and filed for a no-fault divorce, which

is provided for under section 3301(c) of the Divorce Code, 23 Pa.C.S.A. § 3301(c), in March

2015. Roger and Peg remained separated with a pending divorce to the day of Roger’s sudden

death on July 30, 2015. Prior to his death, Roger and Peg agreed to bifurcate the marital

property issues for later resolution as permitted by the Divorce Code. See 23 Pa. C.S.A. §

3323(c.1). They filed their signed affidavits of consent for the no-fault divorce before Roger’s

death. The effect of the filing of signed affidavits of consent established “grounds for the

divorce,” which resulted in the invalidation of the provisions of Roger’s will as they relate to

Peg. See 20 Pa. C.S.A. § 2507(2)(ii). Thus, the court will likely find that Roger’s will

provisions for Peg, both her 1/3 share of Roger’s estate and her appointment as executrix, were

no longer effective.

2. Roger’s estate would be distributed under Roger’s will with 1/3 share to Peg and 1/3

share to Wanda and 1/3 share to be split evenly, 1/6 each, between Michael’s two

children under Pennsylvania’s Anti-Lapse statute.

The distribution of Roger’s estate set forth in his will provided equal 1/3 shares to Peg,

his wife, Michael, his son, and Wanda, his daughter. Both Michael and Wanda were Roger’s

children from a previous marriage. Assuming, as the question states, that Peg and Roger had not

separated and that their marriage had not been subject to a pending divorce at the time of Roger’s

death, the issue becomes how the estate is to be distributed under the will when one of the

beneficiaries has pre-deceased the testator.

The general rule in the event of a beneficiary of a will who dies before the death of the

testator, when there is no “express or implied intent by the testator to the contrary” and except as

otherwise provided by law, is that the legacy of the deceased beneficiary lapses to the residuary

or to intestacy if there is no other residuary beneficiary. 20 Pa.C.S.A. § 2514(10), (11); In re

Estate of Harper, 975 A.2d 1155, 1159 (Pa. Super. 2009); In re McFerren’s Estate, 365 Pa. 490,

491, 76 A.2d 759, 761 (1950). The PEF Code, however, includes an exception to the lapse rule

which provides in relevant part the following:

A devise or bequest to a child or other issue of the testator or to his brother or

sister or to a child of his brother or sister whether designated by name or as one of

a class shall not lapse if the beneficiary shall fail to survive the testator and shall

leave issue surviving the testator but shall pass to such surviving issue who shall

take per stirpes the share which their deceased ancestor would have taken had he

survived the testator.

20 Pa.C.S.A. § 2514(9). This is known as the “anti-lapse statute,” see In re Estate of Harper,

975 A.2d at 1160, which benefits issue of a predeceased child or issue of a brother or sister of the

testator, but not the spouse or any other heir of such pre-deceased beneficiary. Michael was “a

child of the testator,” Roger, and Michael had two children, his “issue,” who survived both

Michael and Roger. Roger’s bequest of 1/3 of his estate to Michael, thus, would pass to

Michael’s surviving children per stirpes. Per stirpes distribution provides that “particular

4

Page 9: JULY 2016 PENNSYLVANIA BAR EXAMINATION · JULY 2016 PENNSYLVANIA BAR EXAMINATION Essay Questions and Examiners’ Analyses and Performance Test Pennsylvania Board of Law Examiners

descendants take among themselves the share of their deceased parent. Persons who take per

stirpes do so in a representative capacity and, standing in the place of their deceased ancestor,

take only what the ancestor would have taken had he or she lived.” Glenda K. Harnad, Kimberly

C. Simmons, 31 Standard Pennsylvania Practice 2d § 150:128 (West Pub. Co. 2016).

This means that Michael’s children will take the share Michael would have taken had he

survived Roger. Because Michael’s two minor children are equal in the line of descent of Roger,

there will be no other beneficiaries of “per stirpes” distribution, as there are no issue in multiple

levels of descent or consanguinity. Therefore each of Michael’s children take by representation

an equal one-half share of Michael’s 1/3 portion. Nothing in the language of Roger’s will

indicated any intent to override the anti-lapse statute, which statute is recognized as having “[t]he

purpose . . . [of] reduc[ing] the incidence of lapses, which are generally disfavored in the law.”

In re Estate of Harper, 975 A.2d at 1160. The net individual estate of Roger, therefore, is

distributed 1/3 to Peg, 1/3 to Wanda, and 1/6 to each of Michael’s two minor children.

3. Both Roger and Larry would be required to declare $2,000 as barter income in

2015. Larry likely could deduct the entire amount he provided through legal

services as a business expense, but Roger’s final return would not have a deduction

for the cleaning services provided to Larry as Larry’s services to Roger for Roger’s

divorce are personal expenses and not business expenses.

This question involves non-cash transactions traded by Roger and Larry in the context of

income and deductions from income under the Internal Revenue Code (IRC).

The IRC defines “gross income” in relevant part as “all income from whatever source

derived.” 26 U.S.C. § 61 (a). Within that definition is the concept of exchanges of goods or

services for which no cash is paid. This is generally referred to as “bartering” or “payments in

kind.” See Rev. Rul. 79-24, 1979-1 C.B. 60; I.R.S., Taxable and Nontaxable Income, Pub. 525

(2015). The regulations governing such transactions are found in the Code of Federal

Regulations (CFR), which state in part as follows:

If services are paid for in exchange for other services, the fair market value of

such other services taken in payment must be included in income as

compensation. If the services are rendered at a stipulated price, such price will be

presumed to be the fair market value of the compensation received in the absence

of evidence to the contrary.

26 CFR §1.61-2(d)(1). According to a ruling by the Internal Revenue Service, in the context of

an illustration of a “barter club,” where a lawyer bartered services with a housepainter, “the fair

market value of services received in exchange for services rendered” must be included as

income. Rev. Rul. 79-24, 1979-1 C.B. 60. This is consistent with the long-standing rationale of

Old Colony Trust Company v Commissioner, 279 U.S 716, 729 (1929) where the Court

determined that gain derived from labor is income regardless of the form of payment.

The facts state that the services rendered by Larry and Roger in 2015 were each worth

$2,000, and Larry and Roger acknowledged that each of the services rendered to the other in

5

Page 10: JULY 2016 PENNSYLVANIA BAR EXAMINATION · JULY 2016 PENNSYLVANIA BAR EXAMINATION Essay Questions and Examiners’ Analyses and Performance Test Pennsylvania Board of Law Examiners

calendar year 2015 were equivalent in value, i.e. $2,000. There is nothing in the facts to indicate

that the fair market value of the services exchanged was anything other than the agreed upon

value for their services. The $2,000 in services that each received for their services is, therefore,

considered income and must be reported by both on their respective federal tax returns.1

The IRC provides that “[t]here shall be allowed as a deduction all the ordinary and

necessary expenses paid or incurred during the taxable year in carrying on any trade or business,

including -- (1) a reasonable allowance for salaries or other compensation for personal services

actually rendered.” 26 U.S.C. § 162(a). The Supreme Court, in Welch v Helvering 290 U.S.

111, (1933), defined “ordinary” and “necessary” business expenses in terms which continue to

be referenced in relevant case law, regulations, and Internal Revenue publications, which are

provided as informal guidance to the general public. The Welch Court stated that while an

expense may be necessary, it also must be ordinary in order to determine whether those expenses

are deductible as “ordinary and necessary expenses.” Id. at 113. The Court indicated that for

such an expense to be considered to be “ordinary and necessary” it would be one that is common

and accepted in the industry. Id. at 114. It also stated that “what is ordinary . . . is none the less

a variable affected by time and place and circumstance.” Id. at 113. In a publication, the

Internal Revenue Service defines “ordinary and necessary” business expense as:

An ordinary expense is one that is common and accepted in your industry. A

necessary expense is one that is helpful and appropriate for your trade or business.

An expense does not have to be indispensable to be considered necessary.

IRS Publication 535 Business Expenses (2015). Cleaning services in a professional office likely

would fit under “ordinary and necessary” expenses. A professional office does ordinarily need

to be cleaned to maintain a professional appearance and the expense of hiring a cleaning service

to do so is likely a necessary expense. This would be similar to an expenditure for landscaping

services “when the expenses legitimately are connected to the taxpayer’s trade or business and

the requirements for deductibility are . . . met.” See Dobbe v Commissioner, 330 T.C. 22 (2000).

In exchange for his legal services, Larry received “personal services actually rendered” by Roger

for the cleaning of his office, which was a location in which he carried on his law practice. Thus,

the $2,000 value of Larry’s legal services which were paid in exchange for Roger’s cleaning

services would likely be deductible for Larry.

Roger, however, provided his cleaning services entirely to pay for the costs of Larry’s

professional legal services provided to Roger for his divorce in 2015. The facts state that none of

Larry’s legal services in 2015 were directed to any of Roger’s business interests. Legal fees paid

for divorce have been determined to be “personal” services not eligible for deduction. Boyer v.

Commissioner, 69 T.C. 521 (1977), citing U. S. v Gilmore, 372 U.S. 39 (1963). The Gilmore

decision resolved a conflict of views in the lower courts regarding the deductibility of legal

expenses which may affect the taxpayer, stating the following:

[T]he origin and character of the claim with respect to which an expense was

incurred, rather than its potential consequences upon the fortunes of the taxpayer,

1 Roger’s return would be filed by his personal representative in his estate as a final return. See I.R.S. Pub.

559, Survivors, Executors, and Administrators (2015).

6

Page 11: JULY 2016 PENNSYLVANIA BAR EXAMINATION · JULY 2016 PENNSYLVANIA BAR EXAMINATION Essay Questions and Examiners’ Analyses and Performance Test Pennsylvania Board of Law Examiners

is the controlling basic test of whether the expense was ‘business’ or ‘personal’

and hence whether it is deductible or not.

Gilmore, 372 U.S. at 49. The value of cleaning services provided by Roger in exchange for

Larry’s legal services to Roger of $2,000 for Roger’s divorce would be characterized as personal

and not ordinary and necessary business expenses. Therefore, Roger would not be able to deduct

any part of the $2,000 value of his cleaning fees, which Roger bartered to Larry, to pay the costs

of his personal divorce.

4. Angela was in violation of Pennsylvania Rule of Professional Conduct 1.8 when she

accepted Sam’s supplemental payments on behalf of Peg without obtaining Peg’s

informed consent.

The payment of legal fees by a third party without the knowledge and approval of the

client is a conflict of interest under the Pennsylvania Rules of Professional Conduct (RPC).

Specifically, RPC 1.8(f) states, in relevant part, as follows:

A lawyer shall not accept compensation for representing a client from one other

than the client unless:

(1) the client gives informed consent;

(2) there is no interference with the lawyer’s independence of professional

judgment or with the client-lawyer relationship; and

(3) information relating to representation of a client is protected as

required by Rule 1.6.

The word “and” previous to the third condition establishes that the conditions are

cumulative, and all three must be met in order for a third party payment to a lawyer to be

permissible under the rule. The narrative clearly indicates, however, that the first condition,

informed consent, was not met. Under the facts, Sam, a third party, asked Angela to charge Peg

a reduced fee and that he would pay any additional amount. He also told Angela not to inform

Peg of the deal. Angela complied and never informed her client, Peg, that Sam was paying

substantial funds to Angela for Peg’s legal fees in the divorce. Angela also made no attempt to

get Peg’s consent to the arrangement. Consequently, Angela violated RPC 1.8 when she

represented Peg in the divorce and permitted Sam to pay part of the fee without informing Peg

and getting Peg’s consent.

7

Page 12: JULY 2016 PENNSYLVANIA BAR EXAMINATION · JULY 2016 PENNSYLVANIA BAR EXAMINATION Essay Questions and Examiners’ Analyses and Performance Test Pennsylvania Board of Law Examiners

Question No. 1: Grading Guidelines

1. Wills-Pending divorce affecting spouse provisions in will

Comments: Candidates should recognize that divorce voids will provisions in favor of a

surviving spouse and that signed and filed affidavits of consent establish ground for divorce

having the same effect on provisions in a testator’s will that favor the surviving spouse as a fully

executed divorce.

6 points

2. Anti-Lapse provision for predeceased issue of testator

Comments: Candidates should recognize the issue of lapse when a will beneficiary predeceases

the testator, and the application of the Pennsylvania Anti-Lapse statute.

5 points

3. Federal Income Tax barter income and business deduction

Comments: Candidates should recognize non-cash transactions such as exchanges of services as

income under the Internal Revenue Code, and that the value of services expended may be

deducted only for ordinary and necessary business purposes.

6 points

4. Professional Responsibility- third party payment to lawyer

Comments: Candidates should recognize that a third party paying the legal fees for a client must

be made known to and approved by the client, and the lawyer’s obligation is to the client.

3 points

8

Page 13: JULY 2016 PENNSYLVANIA BAR EXAMINATION · JULY 2016 PENNSYLVANIA BAR EXAMINATION Essay Questions and Examiners’ Analyses and Performance Test Pennsylvania Board of Law Examiners

Question No. 2

In September 2015, Barb purchased a new vent-free, gas fireplace for her residence in Z

County, Pennsylvania, from Home Hearth, a local retail store. The fireplace was delivered,

assembled, and installed at Barb’s residence by Home Hearth employees. The fireplace was

made and distributed by Friendly Fires, Inc. (FF), a Pennsylvania manufacturer of consumer

heating products, and it was not modified after it left the factory. The fireplace unit was wall

mounted and contained a three-piece artificial log assembly. The log pieces rested on each other

using two interlocking 3/8-inch pins that fit into corresponding indentations on the other pieces

and were positioned over the gas flame to simulate a wood fire when lit. Barb used the fireplace

for three months without incident.

In December 2015, she contacted Ray, proprietor of “Ray’s FireCare,” to service the unit

at her home as FF instructions recommended be done every three months. Ray properly serviced

and positioned the logs over the gas flame opening. A week later, Barb properly turned on the

unit. Two hours later, Barb was discovered lying unconscious in the room with the gas fireplace.

Her breathing was labored, and the room was filled with noxious smoke. She was transported to

the hospital, admitted, and treated for smoke inhalation. After Barb was released, she continued

medical treatment for respiratory difficulties caused by the incident.

Barb contacted FF about the malfunction, but FF refused to speak with her. Barb then

contacted Gary, a manufacturer-certified technician, to inspect the fireplace. Gary had nine

years’ experience examining problems for vent-free fireplace units made by FF and other

manufacturers. Through his examination of the log and the charring patterns, using the usual and

generally accepted methods in fireplace safety, Gary concluded that the logs were not properly

interlocked because a pin intended to hold the logs in place over the flame had broken, causing

9

Page 14: JULY 2016 PENNSYLVANIA BAR EXAMINATION · JULY 2016 PENNSYLVANIA BAR EXAMINATION Essay Questions and Examiners’ Analyses and Performance Test Pennsylvania Board of Law Examiners

misalignment of the log assembly; as a result, the flame burned the logs’ untreated decorative

surfaces rather than their fire-resistant bases and this caused the smoke that injured Barb.

Gary knew, as did FF, that this was a common problem with the design of this unit. The

pins’ size permitted them to be easily broken during routine use, creating an improper fit of the

assembly over the flame and the consequent burning of untreated surfaces that created smoke

and soot. The cost to change the production process and provide a more stable pin was

calculated to be less than $1.50 per unit and the change would not make the unit less effective.

Nevertheless, FF continued to produce and distribute the units, which sell for $900, without any

changes to the pins.

Barb properly and timely filed a complaint including a notice to defend in Z County court

against FF, Home Hearth, and Ray for damages for her personal injury and property loss of her

household items and demanded a jury trial.

1. Barb contends that the log assembly was defectively designed. In addition to

potential negligence or breach of warranty claims under the sales contract or

UCC, what cause of action should Barb bring against FF and Home Hearth based

on her contention that the log assembly was defectively designed and with what

likelihood of success?

2. The smoke and soot from this incident damaged Barb’s carpeting, furniture,

custom-made draperies, and other household property beyond repair. Barb has

obtained estimates that replacing these items will cost $35,000. None of the items

had any special value. If Barb proves that FF or Home Hearth are liable, what

damages would be recoverable by Barb with respect to this property?

3. Barb’s counsel intends to call Gary to testify to his opinion that the broken pin

caused the misalignment of the logs and that the burning of the non-fire resistant

surfaces of the logs caused the smoke. What does Barb’s counsel need to establish

in order to qualify Gary to testify to these matters and with what likely result?

4. Ray’s business is closed, and six months have elapsed after timely and proper

service of the complaint, which alleged that Ray was negligent in his setting of

the logs. He has filed no response. What action should Barb’s counsel take to

obtain a judgment against Ray and with what likely result?

10

Page 15: JULY 2016 PENNSYLVANIA BAR EXAMINATION · JULY 2016 PENNSYLVANIA BAR EXAMINATION Essay Questions and Examiners’ Analyses and Performance Test Pennsylvania Board of Law Examiners

11

Question No. 2: Examiner’s Analysis

1. Barb should file a strict products liability action against Friendly Fires and Home

Hearth on the basis of a design defect.

Section 402A of the Restatement (Second) of Torts was adopted as the law of

Pennsylvania by the Supreme Court in Webb v. Zern, 422 Pa. 424, 220 A.2d 853 (1966). This

section imposes liability on the manufacturer and seller of a defective product for harm caused

thereby regardless of the lack of a contractual relationship between the parties. Salvador v.

Atlantic Steel Boiler Co., 457 Pa. 24, 319 A.2d 903 (1974). The rationale for adoption of the rule

is that sellers and manufacturers are “best able to shoulder the cost[] and . . . administer the risks

involved” with those products released into the stream of commerce. Walton v. Avco Corp., 530

Pa. 568, 575, 610 A.2d 454, 458 (1992).

Section 402(A) provides as follows:

(1) One who sells any product in a defective condition unreasonably dangerous to

the user or consumer or to his property is subject to liability for physical harm

thereby caused to the ultimate user or consumer, or to his property, if

(a) the seller is engaged in the business of selling such a product, and

(b) it is expected to and does reach the user or consumer without

substantial change in the condition in which it is sold.

(2) The rule stated in Subsection (1) applies although

(a) the seller has exercised all possible care in the preparation and sale of

his product, and

(b) the user or consumer has not bought the product from or entered into

any contractual relation with the seller.

Restatement (Second) of Torts, § 402A.

In this case FF manufactured and sold the fireplace to Home Hearth who then sold it to

Barb. Thus, FF was engaged in selling the fireplace.

To prevail on a products liability claim under Section 402A, “‘the plaintiff must [also]

prove (1) that the product was defective, (2) that the defect existed when it left the [control] of

the defendant, and (3) that the defect caused the harm.’” Putt v. Yates-American Mach. Co., 722

A.2d 217, 220 (Pa.Super. 1998) (quoting Riley v. Warren Mfg. Inc, 455 Pa. Super. 384, 688 A.2d

221, 224 (1997)). “[T]he plaintiff may show that the product was defective due to a design

defect, i.e. that the design of the machine resulted in an unreasonably dangerous product.” Id. at

221. In a case alleging a design defect, “the question is whether the product should have been

Page 16: JULY 2016 PENNSYLVANIA BAR EXAMINATION · JULY 2016 PENNSYLVANIA BAR EXAMINATION Essay Questions and Examiners’ Analyses and Performance Test Pennsylvania Board of Law Examiners

designed more safely.” Dambacher v. Mallis, 336 Pa.Super. 22, 57, 485 A.2d 408, 426 (1984),

appeal denied, 508 Pa. 643, 500 A.2d 428 (1985).

The Pennsylvania Supreme Court in Tincher v. Omega Flex, Inc., 104 A.3d 328 (Pa.

2014) has recently articulated the analysis for determining whether a product is defective in the

context of a design defect with respect to strict product liability claims. In Tincher the Court

reaffirmed the viability of a “properly calibrated” Restatement (Second) of Torts, § 402A,

rejected adoption of the Restatement (Third) of Torts, and outlined a new “composite standard”

for strict product liability analysis. Tincher, 104 A.3d at 399, 401. The court held that a plaintiff

can demonstrate that a product contains a “defective condition” in the nature of a design defect

through either or both of the following: (1) the “consumer expectations test,” or (2) the “risk-

utility test.” Tincher, 104 A.3d at 401.

Under the first element of a strict products liability claim a plaintiff must prove that the

product was defective using the “consumer expectations” test or the “risk utility” test. The

“consumer expectations” test provides that a “product is in a defective condition if the danger is

unknowable and unacceptable to the average or ordinary consumer.” Tincher, 104 A.3d at 387.

A “product is not defective if the ordinary consumer would reasonably anticipate and appreciate

the dangerous condition of the product and the attendant risk of injury of which the plaintiff

complains.” Id. Here the danger from smoke caused by the burning of the untreated portion of

the logs, which Barb alleges was caused by the breaking off of the small pin crucial to proper

alignment, would be unknowable and unacceptable to the average consumer. The logs were

intended to be used and were used and positioned over an open flame in the fireplace. The logs

had an untreated surface that caused noxious smoke when burned, which was likely to cause

injury to a consumer or a consumer’s property. A consumer, here Barb, could not reasonably be

expected to know that the small pin could easily break and cause the logs to be misaligned and

consequently burn the untreated surface causing noxious smoke. This would likely be sufficient

to meet the first element of the defective design cause of action.

A similar conclusion is also reached under the “risk-utility” test. This test provides that

“a product is in a defective condition if a ‘reasonable person’ would conclude that the probability

and seriousness of harm caused by the product outweigh the burden or costs of taking

precautions.” Id.at 389. In other words the “risk-utility” test involves an evaluation of “whether

a manufacturer’s conduct in manufacturing or designing a product was reasonable.” Id. The

Tincher Court discusses some possible factors in applying the risk-utility test including the

product’s usefulness, the likelihood the product will cause injury and the seriousness of the

potential injury, the availability of an alternative or substitute product, the manufacturer’s ability

to eliminate the harmful aspect of the product without limiting its usefulness, the user’s ability to

avoid harm through the use of due care, the user’s awareness of the dangers through the

product’s condition or warnings, and the likelihood of loss spreading through insurance or

otherwise. Tincher, 104 A.3d at 389-90. In this case both Gary and FF knew that the small pin

breaking was a common problem and the burden of taking precautions, i.e. redesigning the pin to

make it sturdier, was minimal at $1.50 per fireplace. Additionally, the likelihood that the design

would cause harm was high as the log assembly was to be placed over a flame, the pin was likely

to break causing an untreated part of the log to burn, and noxious smoke would result. The facts

also demonstrate that the usefulness of the fireplace would not have been limited if FF had

12

Page 17: JULY 2016 PENNSYLVANIA BAR EXAMINATION · JULY 2016 PENNSYLVANIA BAR EXAMINATION Essay Questions and Examiners’ Analyses and Performance Test Pennsylvania Board of Law Examiners

manufactured the unit with a sturdier pin. Moreover, there was no way for Barb to be aware of

the danger by looking at the product or through warning and the risk of loss is high given the

resulting noxious smoke and soot. The probability and seriousness of the harm that could be

caused by the smoke outweighs the small burden of correcting the problem. Accordingly, it is

likely that the trier of fact would find that the product meets the first element of a defective

design cause of action under either the risk/utility test or the consumer expectations test.

The second element of the defective design cause of action, that the defect existed at the

time the product left FF’s control, is easily met. The facts show that Barb bought the fireplace

unit from Home Hearth and that it was manufactured by FF. The facts also show the fireplace

unit was not modified after leaving FF’s factory. Thus, the second element of the defective

design cause of action is met.

In addition, the third element of the defective design cause of action, that the defect was

the proximate and actual cause of Barb’s injury, is met. The facts state that Gary concluded that

the logs were not properly interlocked because a pin intended to hold the logs in place over the

flame had broken, causing misalignment of the log assembly and that as a result, the flame

burned the logs’ untreated decorative surfaces rather than their fire-resistant bases, and that this

caused the smoke which injured Barb and damaged her property. Additionally, Barb was

discovered lying unconscious with labored breathing in the room, which was filled with noxious

smoke. She was transported to the hospital, admitted, and treated for smoke inhalation. In

addition, the facts also provide that the smoke and soot from this incident damaged the carpeting,

furniture, custom-made draperies, and other household property beyond repair. Thus, the smoke

and soot resulting from this defective design was the proximate and actual cause of Barb’s

personal injury and property damages and establishes the third element of this tort.

Barb can also maintain a cause of action for strict products liability on the same grounds

of design defect against Home Hearth. A seller that engages in the business of supplying a

product which may endanger the safety of persons or property undertakes a special responsibility

for the safety of the public using the product. Francioni v. Gibsonia Truck Corp., 472 Pa. 362,

366-67, 372 A.2d 736, 738 (1977). Liability under strict products liability law applies to all

suppliers that engage in the business of selling defective products irrespective of fault on the part

of that supplier. Moran v. G. & W.H. Corson, Inc. 402 Pa.Super. 101, 122, 586 A.2d 416, 427

(1991) allocatur denied, 529 Pa. 650, 602 A.2d 860 (1992); see also Restatement (Second) of

Torts, Section 402A(1)(a). As applied to these facts, Home Hearth is the seller of the vent-free

fireplaces to the public. Home Hearth engaged in selling the Friendly Fires’ product to the

consuming public. Its potential lack of knowledge of the defect is irrelevant and Home Hearth

can also be held liable for a claim by Barb because it sold her the defective Friendly Fires’ unit.

2. Barb would be able to recover the actual value of her destroyed personal property

valued at the time it was destroyed and not its replacement value.

“Where there is a total destruction, the measure of damages ‘would be the actual value’

of the property itself, 'taking into consideration its age, condition, and any other circumstances

affecting it, and less anything salvaged from it.” Russell et al. v. U.S., 113 F. Supp. 353, 356

(M.D. Pa. 1953), citing Jones v. Monroe Electric Co., 350 Pa. 539, 544, 39 A.2d 569, 571

13

Page 18: JULY 2016 PENNSYLVANIA BAR EXAMINATION · JULY 2016 PENNSYLVANIA BAR EXAMINATION Essay Questions and Examiners’ Analyses and Performance Test Pennsylvania Board of Law Examiners

(1944). In this case the facts state that Barb’s household property has been damaged beyond

repair and has no special value. Notwithstanding the fact that the full replacement cost for her

property is $35,000, the measure of damages for total destruction of personal property under a

strict products liability claim does not entitle her to the cost of replacement value of the items.

Damages will be assessed based upon the actual value of Barb’s personal property at the time of

destruction, considering its condition and reasonable value to a third party at that time.

3. Barb’s attorney should establish Gary’s credentials showing that he is qualified to

testify as an expert based on his knowledge, skill, training, and experience. The

court will likely agree and find that Gary is qualified to testify as to the cause of the

smoke.

Rule 702 of the Pennsylvania Rules of Evidence is applicable to expert testimony. That

rule provides the following:

A witness who is qualified as an expert by knowledge, skill, experience, training,

or education may testify in the form of an opinion or otherwise if:

(a) the expert’s scientific, technical, or other specialized knowledge is

beyond that possessed by the average layperson;

(b) the expert’s scientific, technical, or other specialized knowledge

will help the trier of fact to understand the evidence or to

determine a fact in issue; and

(c) the expert’s methodology is generally accepted in the relevant field.

Pa.R.E. No. 702.

In Miller v. Brass Rail Tavern, Inc., 541 Pa. 474, 664 A.2d 525 (1995), the Pennsylvania

Supreme Court stated the rule for qualifying a witness to testify as an expert as follows:

The test to be applied when qualifying an expert witness is whether the witness

has any reasonable pretension to specialized knowledge on the subject under

investigation. If he does, he may testify and the weight to be given to such

testimony is for the trier of fact to determine.

Miller, 541 Pa. at 480-81, 664 A.2d at 528. Although Rule 702 was adopted after Miller was

decided, its adoption did not change the standard set out in Miller. Pa.R.E. Rule 702 cmt.

“Expert testimony is admissible in all cases, civil and criminal alike, when it involves

explanations and inferences not within the range of ordinary training, knowledge, intelligence

and experience.” Commonwealth v. Leslie, 424 Pa. 331, 334, 227 A.2d 900, 903 (1967). On the

facts presented, Gary is a certified technician with over nine years of experience examining the

vent-free fireplace units produced by Friendly Fires and other manufacturers. Gary will likely be

14

Page 19: JULY 2016 PENNSYLVANIA BAR EXAMINATION · JULY 2016 PENNSYLVANIA BAR EXAMINATION Essay Questions and Examiners’ Analyses and Performance Test Pennsylvania Board of Law Examiners

qualified based on training and experience. He has conducted examinations of units and can be

instructive to the jury in this case.

His testimony regarding the location of the charring marking on the logs and the broken

pin based on his use of recognized techniques would likely be helpful to the fact-finder in

understanding the misalignment of the logs and the cause of the smoke and soot which Barb

alleges caused her injury and property damage. To be admissible under Pa.R.E. 702, proffered

expert testimony must also address matters “beyond the knowledge possessed by [the average]

layperson.” Commonwealth v. Walker, 625 Pa. 450, 473, 92 A.3d 766, 780 (2014). Gary’s

testimony regarding his analysis of the burn pattern and the markings as being caused by the

improper alignment of the logs would also likely be beyond the knowledge of a layperson

examining the physical evidence. Additionally, the facts also provide that Gary examined the

fireplace and charring patterns using the usual and generally accepted methods in fireplace

safety. Therefore, given Gary’s extensive history and experience with vent-free fireplaces and

his expertise in assessing issues with same, the court should rule that Gary’s testimony as an

expert is admissible.

4. Barb’s counsel should seek to obtain a default judgment against Ray, which will

likely be granted.

The facts state that Ray has been properly served with Barb’s complaint which contained

a proper notice to defend and that 6 months have elapsed without Ray filing a response.

Pa.R.C.P. No. 1026 provides in pertinent part as follows:

Time for Filing. Notice to Plead.

(a) [E]very pleading subsequent to the complaint shall be filed within twenty

days after service of the preceding pleading, but no pleading need be filed

unless the preceding pleading contains a notice to defend or is endorsed

with a notice to plead.

Thus Ray’s responsive pleading1 was due within 20 days of Barb’s service of the complaint,

which contained a proper notice to defend.

Based upon Ray’s failure to respond to the complaint, Barb’s counsel should seek a

default judgment against Ray under the Pennsylvania Rules of Civil Procedure. Pa.R.C.P. No.

1037 provides in pertinent part the following:

(b) The prothonotary, on praecipe of the plaintiff, shall enter judgment against

the defendant for failure to file within the required time a pleading to a

complaint which contains a notice to defend or, except as provided by

subdivision (d), for any relief admitted to be due by the defendant’s

pleadings.

1 Pleadings consist of a complaint, an answer, a reply, a counter-reply, and preliminary objections. Pa.R.C.P. No.

1017.

15

Page 20: JULY 2016 PENNSYLVANIA BAR EXAMINATION · JULY 2016 PENNSYLVANIA BAR EXAMINATION Essay Questions and Examiners’ Analyses and Performance Test Pennsylvania Board of Law Examiners

***

(1) The prothonotary shall assess damages for the amount to which the

plaintiff is entitled if it is a sum certain or which can be made

certain by computation, but if it is not, the damages shall be

assessed at a trial at which the issues shall be limited to the amount

of the damages.

***

(c) In all cases, the court, on motion of a party, may enter an appropriate

judgment against a party upon default or admission.

The facts state that Barb timely and properly served Ray with the complaint which

contained a proper notice to defend. Ray failed to respond within the 20 day time period set

forth in the Rules of Civil Procedure, therefore Barb’s counsel can proceed under Pa.R.C.P. No.

1037 to seek a default judgment.

In order to pursue a default judgment, Barb must send Ray a written notice of intention to

file a praecipe to enter judgment by default at least ten days prior to filing the praecipe for

default judgment with the prothonotary. Pa.R.C.P. No. 237.1. Rule 237.1 requires that the

praecipe for default judgment contain a certification that the party seeking a default judgment

sent written notice of his or her intent to file the praecipe for default judgment to the defendant.

After ten days have elapsed from the time the notice of praecipe to enter judgment by default is

sent, if Ray still fails to file a response to the complaint Barb’s attorney may file a praecipe to

enter judgment by default against Ray attaching a copy of the notice of praecipe to enter

judgment by default. The prothonotary likely will enter judgment by default against Ray and in

favor of Barb under Pa.R.C.P. No. 1037(b).

After the entry of the default judgment, Barb may seek an assessment of damages under

Rule 1037. If, as they are here, damages are not a “sum certain,” as this action involves a claim

for both personal injuries and property damage, Barb can proceed to a trial with the evidence

limited to her damages. King v. Fayette Aviation, 226 Pa.Super. 588, 323 A.2d 286 (1974).

16

Page 21: JULY 2016 PENNSYLVANIA BAR EXAMINATION · JULY 2016 PENNSYLVANIA BAR EXAMINATION Essay Questions and Examiners’ Analyses and Performance Test Pennsylvania Board of Law Examiners

Question No. 2: Grading Guidelines

1. Torts – Strict Products Liability

Comments: The candidate should recognize that Barb may assert a cause of action for strict

products liability on the basis of a design defect against Friendly Fires and Home Hearth, state

the elements required to prove liability, apply the facts to the elements, and conclude that she has

a strong likelihood of success.

8 Points

2. Damages – Personal Property

Comments: The candidate should state the measure of damages when personal property and

which has no special value is destroyed and conclude that Barb can recover only the “actual

value” of her household property and not the replacement value.

2 Points

3. Evidence – Expert Testimony

Comments: The candidate should recognize that the Pennsylvania Rules of Evidence permit a

qualified person to testify as an expert, discuss the requirements as applied to the facts and

conclude that the court will permit Gary to testify as an expert.

6 Points

4. Civil Procedure – Default Judgment

Comments: The candidate should recognize that Ray’s failure to respond after being served with

a complaint containing a proper notice to defend permits Barb to serve Ray with a notice of

intent to seek a default judgment and after 10 days proceed to obtain a default judgment against

Ray under the Pennsylvania Rules of Civil Procedure and to proceed to trial against him limited

only to the issue of damages.

4 Points

17

Page 22: JULY 2016 PENNSYLVANIA BAR EXAMINATION · JULY 2016 PENNSYLVANIA BAR EXAMINATION Essay Questions and Examiners’ Analyses and Performance Test Pennsylvania Board of Law Examiners

Question No. 3

Rachel and Josh were married in C County, Pennsylvania, in April of 2012, and shortly

thereafter they purchased a home together on 15 Mockingbird Lane in that county. Two weeks

before the wedding, Josh presented Rachel with a prenuptial agreement that fully and accurately

disclosed that he had a total of $800,000 in assets and that, in the event of divorce, he would be

entitled to all of those assets including any increased value. Rachel wanted to hire a lawyer to

review the agreement but could not afford one. She proceeded to carefully review the

agreement, understood it, and voluntarily signed it two days before the wedding.

In September 2015, Josh suspected that Rachel was having an affair with Pat. Josh

decided to beat up Pat to send a message that he should leave Rachel alone. Josh proceeded to

the office where Pat worked, and waited for him. When Pat left work, Josh approached him and

beat him about the head and torso with a 2x4 piece of lumber that was lying nearby. When Josh

was done, he said to Pat, “Think twice next time about spending time with Rachel.”

After the beating, Josh fled the scene in his SUV. Unbeknownst to Josh, Marissa was

standing in the shadows and witnessed the beating and overheard what was said. She

immediately called the police and gave a description of Josh, the 2x4 he used, and the SUV. The

police and ambulance personnel responded to the scene, and Pat was rushed to the hospital. Pat

died later that evening from the injuries sustained in the beating.

The next day, Josh attended a bazaar and saw a priest, later identified as Father John.

Josh, who was not a member of any religious faith and didn’t know Father John, approached him

and said, “Father, I did something really bad yesterday to Pat, but he deserved it. He should have

just left Rachel alone.” When Father John read the paper the next day, he saw that Pat was the

victim of a recent attack. Father John called the police and reported what Josh told him.

18

Page 23: JULY 2016 PENNSYLVANIA BAR EXAMINATION · JULY 2016 PENNSYLVANIA BAR EXAMINATION Essay Questions and Examiners’ Analyses and Performance Test Pennsylvania Board of Law Examiners

Two days after the beating, a police officer saw an SUV fitting the description given by

Marissa. The officer immediately pulled the vehicle over. Upon approaching the SUV, the

officer observed the driver, who fully and accurately fit the description of the assailant given by

Marissa, and observed apparent blood on the passenger seat. Josh was immediately placed under

arrest. Another officer immediately drove to Josh’s and Rachel’s home and knocked on the

door. Rachel answered the door, identified herself as Josh’s wife, and stated that she lived there

with Josh. She was informed that Josh had just been placed under arrest, and she was asked for

permission to search the home. Rachel understood the request and voluntarily agreed to it. The

police then located a bloody 2x4 on the kitchen counter. The police then obtained a valid

warrant to seize the 2x4. Subsequent DNA testing revealed the blood was a match to Pat. The

police did not attempt to secure a search warrant for the initial search.

1. What degree of murder charge against Josh is most likely supported by the facts?

2. If the defense files an omnibus pretrial motion requesting suppression of the 2x4

based on the right to be free from illegal search of Josh’s home, how should the

district attorney respond, and how would the court likely rule?

3. If the murder charge proceeds to trial and defense counsel tries to prevent Josh’s

statement to Father John from being introduced at trial on the basis of the

clergy/communicant privilege, how should the district attorney respond, and how

would the court likely rule?

After Josh was charged and incarcerated Rachel decided to file for divorce. She decided

to explore a challenge to the validity of the prenuptial agreement on the grounds that it was

fundamentally unfair due to the fact that she did not get a share in any of the increase in value of

Josh’s pre-marital assets, which had increased in value in the uncontested amount of $300,000,

and on the grounds that she didn’t have a lawyer review it.

4. If Rachel comes to Lawyer Jones for advice on whether her proposed challenges

to the validity of the prenuptial agreement would be successful, how should she

advise her?

19

Page 24: JULY 2016 PENNSYLVANIA BAR EXAMINATION · JULY 2016 PENNSYLVANIA BAR EXAMINATION Essay Questions and Examiners’ Analyses and Performance Test Pennsylvania Board of Law Examiners

Question No. 3: Examiner’s Analysis

1. A charge of third degree murder against Josh is most likely supported by the facts

presented.

Under 18 Pa. C.S.A. § 2502(c), third-degree murder encompasses all forms of

murder which do not constitute first-degree murder (intentional killing) or second-

degree murder (killing committed during the perpetration of a felony). In the

context of third-degree murder, the Commonwealth need not establish a specific

intent to kill, or even a specific intent to harm the victim. The Commonwealth

need only establish a killing with malice, i.e., the death of another brought about

by an intentional act which indicates a wickedness of disposition, hardness of

heart, wantonness, cruelty, recklessness of consequences, or a mind lacking

regard for social duty.

Commonwealth v. Johnson, 719 A.2d 778, 785 (Pa. Super. 1998) (citation omitted). Malice is

established where the defendant’s intentional act indicates that the defendant “‘consciously

disregard[ed] an unjustified and extremely high risk that his actions might cause death or serious

bodily [injury].’” Commonwealth v. Seibert, 622 A.2d 361, 364 (Pa. Super. 1993) (quoting

Commonwealth v. Young, 494 Pa. 224, 228, 431 A.2d 230, 232 (1981)). Malice may be inferred

from all of the circumstances surrounding the defendant’s conduct, and is properly inferred from

the use of a deadly weapon on a vital party of the body. Commonwealth v. Cruz-Centeno, 668

A.2d 536, 540 (Pa. Super. 1995). In Commonwealth v. Marks, 704 A.2d 1095, 1100 (Pa. Super.

1997), the court concluded that the evidence demonstrated and the conviction of third degree

murder was supported by the fact that the defendant used a sledgehammer to repeatedly hit the

victim producing 12 lacerations to the top, back, and side of the victim’s head.

“A criminal homicide constitutes murder of the first degree when it is committed by an

intentional killing.” 18 Pa. C.S.A. § 2502(a). An intentional killing is defined as a “[k]illing by

means of poison, or by lying in wait, or by any other kind of willful, deliberate and premeditated

killing.” 18 Pa. C.S.A. § 2502(d). Murder of the second degree is “[a] criminal homicide . . .

committed while the defendant was engaged as a principal or an accomplice in the perpetration

of a felony.” 18 Pa. C.S.A. § 2502(b). Perpetration of a felony is defined as “[t]he act of the

defendant in engaging in or being an accomplice in the commission of, or an attempt to commit,

or flight after committing, or attempting to commit robbery, rape, or deviate sexual intercourse

by force or threat of force, arson, burglary or kidnapping.” 18 Pa. C.S.A. § 2502(d).

As applied here, the facts clearly indicate that Josh set out to beat up Pat in order to send

a message that Pat should leave Rachel alone. Josh used the 2x4, which he found at the scene, to

repeatedly beat Pat about the head and torso; the malice necessary to establish third-degree

murder can be inferred from the use of the 2x4 on a vital part of Pat’s body, in particular, his

head. After the beating, Josh said to Pat, “Think twice next time about spending time with

Rachel.” When reviewing all of the circumstances of the beating, including the statement made

by Josh to Pat at the conclusion of the beating, it appears that Josh’s intent was to send a message

to Pat to stay away from Rachel and not to kill him. Thus, the specific intent to kill required for

first-degree murder is arguably not supported by the facts presented. Josh’s actions of beating

Pat about the head with a 2x4, demonstrates that he consciously disregarded an unjustified and

20

Page 25: JULY 2016 PENNSYLVANIA BAR EXAMINATION · JULY 2016 PENNSYLVANIA BAR EXAMINATION Essay Questions and Examiners’ Analyses and Performance Test Pennsylvania Board of Law Examiners

extremely high risk that he would cause serious bodily injury or death to Pat. Accordingly,

Josh’s actions likely support the malice necessary for a conviction of third-degree murder as he

demonstrated a wickedness of disposition, hardness of heart, wantonness, cruelty, recklessness of

consequences, or a mind lacking regard for social duty, and that charge is likely supported by the

facts presented.

A charge of second degree murder is clearly not supported by the facts as the killing was

not committed during the perpetration of one of the felonies enumerated in 18 Pa. C.S.A. §

2502(d).

2. The District Attorney should respond that although the police did not have a

warrant to search Josh’s home, they did have consent from a co-owner of the home

which will fall within a recognized exception to the search warrant requirement and

the court will likely rule that the search was lawful.

“[B]oth the Fourth Amendment to the United States Constitution and Article I, § 8 of the

Pennsylvania Constitution prohibit unreasonable searches and seizures.” Commonwealth v.

Harris, 888 A.2d 862, 868 (Pa. Super. 2005); see U.S. Const. amend. IV; Pa. Const. art. I § 8.1

“Although as a general rule, warrantless searches unsupported by probable cause are

unreasonable, [Pennsylvania] courts have recognized an exception when a third party consents to

the search[.]” Commonwealth v. Simmen, 58 A.3d 811, 816 (Pa. Super. 2012). In Simmen, the

court stated as follows:

Both the federal and Pennsylvania constitutions permit third party consent to a

search. When police officers obtain the voluntary consent of a third party who

has the authority to give consent, they are not required to obtain a search warrant

based upon probable cause. The Supreme Court explained that a third party

possessing common authority over a premises can give valid consent to search

against a non-consenting person who shares authority because it is reasonable to

recognize that any of the co-inhabitants has the right to permit the inspection in

his own right and that the others have assumed the risk that one of their number

might permit the common area to be searched.

Id. at 816-17 (quoting Commonwealth v. Reese, 31 A.3d 708, 722 (Pa. Super. 2011) (quoting

Commonwealth v. Hughes, 836 A.2d 893, 900 (2003)). In Commonwealth v. Yancoskie, 915

A.2d 111, 115 (Pa. Super. 2006), the court affirmed the lawfulness of a warrantless search where

defendant’s wife permitted the search of the marital home when the husband was not present,

and the search resulted in the seizure of incriminating evidence.

The district attorney should respond that the police were permitted to search without a

warrant as they had third party consent to search the home. As applied here, it is undisputed that

the police did not have a valid search warrant for the marital residence of Josh and Rachel.

However, the police secured the proper consent from Rachel, who was both Josh’s wife and co-

owner of the premises, to conduct the search. It appears that Rachel’s consent was voluntary as

the facts provide that she understood the request and voluntarily agreed to the search.

1 Although there are sometimes greater rights under Article I, § 8 of Pennsylvania Constitution than the Fourth

Amendment to United States Constitution, the rights here are the same.

21

Page 26: JULY 2016 PENNSYLVANIA BAR EXAMINATION · JULY 2016 PENNSYLVANIA BAR EXAMINATION Essay Questions and Examiners’ Analyses and Performance Test Pennsylvania Board of Law Examiners

Additionally, as the co-owner and co-tenant she shared authority over the area in common with

Josh. Here the 2x4 was located in the kitchen, which would be deemed to be a common area of

the home. Despite the fact that the police did not secure a search warrant, it appears that the

search would be deemed valid under the legally recognized consent exception to the search

warrant requirement. Therefore, the court will likely rule that the 2x4 is admissible and the

search did not violate Josh’s right to be free from unreasonable searches and seizures under the

Fourth Amendment to the United States Constitution or Article I, § 8 of the Pennsylvania

Constitution.

3. The District Attorney should respond that Josh’s statement to Father John is not

covered by the clergy-communicant privilege and, therefore, Father John should be

permitted to testify to the statement at trial.

In a clergy-communicant relationship, the communicant holds the privilege to prevent the

disclosure of confidential communications made pursuant to that relationship. See 42 Pa.C.S.A.

§ 5943. The clergy/communicant privilege is codified at 42 Pa.C.S.A. § 5943 and provides as

follows:

No clergyman, priest, rabbi, or minister of the gospel of any regularly established

church or religious organization, except clergymen or ministers, who are self-

ordained or who are members of religious organizations in which members other

than the leader thereof are deemed clergymen or ministers, who while in the

course of his duties has acquired information from any person secretly and in

confidence shall be compelled, or allowed without consent of such person, to

disclose that information in any legal proceeding, trial or investigation before any

government unit.

“Pennsylvania courts have interpreted our clergy-communicant privilege as applying only

to confidential communications between a communicant and a member of the clergy in his or her

role as confessor or spiritual counselor.” Commonwealth v. Stewart, 690 A.2d 195, 197-98 (Pa.

1997). In Commonwealth v. Patterson, 572 A.2d 1258, 1262, 1264 (Pa. Super. 1990) the court

was faced with a challenge to the admissibility of statements made by the defendant to a clergy

member. In that case, the defendant made statements to the clergy member in his capacity as a

court appointed counselor and not in a confessional or spiritual role. Thus, the court determined

that “the statements were not motivated by religious considerations or in order to seek the

forgiveness of God.” Id. at 1265. The court concluded that the statements were not made to the

clergy member “in the course of his duties as a minister[,]” but rather as a court appointed

counselor. Id. The court also noted that the defendant was never a member of the clergy

member’s church. The court went on to conclude that the statements made to the clergy member

were not protected by the clergy-communicant privilege. Id.

As applied to this case, Josh approached Father John at a public bazaar. He did not

approach Father John in a confidential or private setting. When Josh approached Father John he

stated that he did something really bad, but that Pat deserved it. He also went on to say that Pat

should have just left Rachel alone. There is no indication that Josh sought to keep this

information confidential or that he was seeking Father John out in his role as a spiritual

counselor or that he was seeking forgiveness for what he had done. Further, Josh was not a

member of Father John’s church. Although Josh may have been entitled to invoke the

22

Page 27: JULY 2016 PENNSYLVANIA BAR EXAMINATION · JULY 2016 PENNSYLVANIA BAR EXAMINATION Essay Questions and Examiners’ Analyses and Performance Test Pennsylvania Board of Law Examiners

protections of the clergy-communicant privilege if he had sought out Father John in private and

in his role as spiritual counselor, he did not do so. Accordingly, the privilege does not appear to

apply in these circumstances. Thus, Father John should be permitted to testify to the statements

made to him by Josh. The court will likely rule in favor of the admission of these statements.

4. Lawyer Jones should advise Rachel that her proposed challenges to the validity of

the prenuptial agreement will likely not be successful under existing law.

“Prenuptial agreement are contracts, and, as such, should be evaluated under the same

criteria as are applicable to other types of contracts. . . . Absent fraud, misrepresentation, or

duress, spouses should be bound by the terms of their agreements.” Simeone v. Simeone, 581

A.2d 162, 165 (Pa. 1990).2 “[T]he reasonableness of a prenuptial bargain is not a proper subject

for judicial review. Id. at 166. “Contracting parties are normally bound by their agreements,

without regard to whether the terms thereof were read and fully understood and irrespective of

whether the agreements embodied reasonable or good bargains.” Id. at 165 (citing Standard

Venetian Blind Co. v. American Empire Insurance Co., 469 A.2d 563, 566 (Pa. 1983)). Further,

in Simeone, the court rejected the argument that an agreement should be declared void on the

grounds that the aggrieved party did not consult with independent legal counsel. Id. at 166. In

reaching its determination, the court declined “[t]o impose a per se requirement that parties

entering a prenuptial agreement must obtain independent legal counsel [as this] would be

contrary to the traditional principles of contract law, and would constitute a paternalistic and

unwarranted interference with the parties’ freedom to enter contracts.” Id.

As applied here, the facts indicate that Josh made a full and accurate disclosure of his

$800,000 in assets when he presented the prenuptial agreement to Rachel. The agreement clearly

provided that in the event of divorce he would be entitled to all of these assets including any

increase in value thereof. In the facts it is stipulated that the increase in value in this case was

$300,000. The facts further indicate that Rachel desired to hire a lawyer to review the agreement

but was unable to afford one. Despite this, the facts clearly indicate that she understood the

agreement and voluntarily elected to sign it. Although it might have been advisable for Rachel

to have a lawyer review the agreement, this will not provide a viable basis to challenge the

agreement under existing law cited above. Further, the fact that the agreement may not be

reasonable is likewise not a valid basis to challenge its validity. The facts clearly indicate that

Rachel understood the terms of the agreement and proceeded to voluntarily execute it, and she

cannot now, after the fact, argue that it was not reasonable. If Rachel felt that the terms of the

agreement were unreasonable, that argument should have been raised before the wedding and

appropriate modifications to the agreement should have been sought at that time. Alternatively,

Rachel could have elected to decline signing the agreement. Raising the argument of

unreasonableness at this juncture will result in an unsuccessful challenge to the prenuptial

agreement.

2 The legislature has enacted a provision governing the standards for challenging the enforceability of premarital

agreements at 23 Pa. C.S.A. § 3106 that encompasses the approach set forth in Simeone.

23

Page 28: JULY 2016 PENNSYLVANIA BAR EXAMINATION · JULY 2016 PENNSYLVANIA BAR EXAMINATION Essay Questions and Examiners’ Analyses and Performance Test Pennsylvania Board of Law Examiners

Question No. 3: Grading Guidelines

1. Third Degree Murder

Comments: The Candidate is expected to recognize that Third Degree Murder is mostly likely

supported by the facts presented and the candidate should discuss the controlling law and

applicable facts.

6 points

2. Consent to Search

Comments: The Candidate should recognize that the District Attorney should respond by

indicating that there is an exception to the search warrant requirement for third party consent and

the candidate should discuss the applicable law and facts relative to this issue and conclude that

the search was likely lawful and the evidence should be admissible.

5 points

3. Clergy-Communicant Privilege

Comments: The Candidate should discuss the law regarding the clergy/ communicant privilege

and apply the applicable facts and conclude that the privilege likely does not apply under the

facts presented.

5 points

4. Prenuptial Agreements

Comments: The Candidate should discuss the applicable legal principles governing the

interpretation of pre-nuptial agreements, apply the applicable facts and conclude that the pre-

nuptial agreement is likely enforceable.

4 points

24

Page 29: JULY 2016 PENNSYLVANIA BAR EXAMINATION · JULY 2016 PENNSYLVANIA BAR EXAMINATION Essay Questions and Examiners’ Analyses and Performance Test Pennsylvania Board of Law Examiners

Question No. 4

Andrew is an African-American man employed as an analyst at A Corp, a regional

company employing 100 people. Andrew completes his projects at A Corp with Tom, his fellow

employee, who is white. Both Andrew and Tom are considered by their boss to be good

employees and both received above-average evaluations. In his spare time, Andrew is involved

with a group called “Peaceful Community” (hereinafter “PC”). PC’s mission is to promote

acceptance of diversity in the community.

Andrew has been tasked by PC to find a place where PC members can hand out

pamphlets describing PC’s message and calendar of upcoming events. One morning, while

walking through Green Park, the only public park in B City, Andrew thought it would be a good

place to hand out PC’s pamphlets because the park was busy with people walking to work and

parents walking children to school. As he was leaving Green Park, Andrew noticed a small sign

posted that said, “All Leafletting Prohibited.” Concerned, Andrew called the director of B City

Parks Department and was told that a B City ordinance prohibits leafletting, meaning the

handing out of pamphlets, of any kind in Green Park. The director told Andrew that the

ordinance banning leafletting was made in an effort to decrease the amount of litter in Green

Park and that violation of the ordinance was punishable by a fine.

Recently, Andrew and Tom missed a big deadline, for which they were jointly

responsible, at A Corp. That missed deadline caused the company to lose a major client, i.e. C

Corp. Andrew and Tom’s boss was incensed at the pair and decided to fire Andrew, A Corp’s

only African-American employee. When Andrew asked his boss why he was being terminated,

his boss said, “We don’t need people like you working here, Andrew.” As Andrew was leaving

A Corp, he slipped on a patch of untreated ice in the parking lot, fracturing his arm.

25

Page 30: JULY 2016 PENNSYLVANIA BAR EXAMINATION · JULY 2016 PENNSYLVANIA BAR EXAMINATION Essay Questions and Examiners’ Analyses and Performance Test Pennsylvania Board of Law Examiners

After exhausting administrative remedies, Andrew filed a lawsuit, within the statutory

time limits, against A Corp in federal court, alleging a Title VII racial discrimination claim based

on his termination and a negligence claim for his slip and fall. In response to the lawsuit, A Corp

has taken the position that the termination was justified because Andrew recently missed an

important deadline that resulted in A Corp losing a major client, C Corp. During discovery, it

was uncovered that, in the six months preceding Andrew’s termination, Andrew’s boss sent

several emails from his work account containing racist jokes to his friends and colleagues.

1. If Andrew and PC file suit against the appropriate parties alleging that B City’s

ordinance prohibiting the distribution of pamphlets in Green Park violates

Andrew’s and PC’s First Amendment right to free speech under the U.S.

Constitution, based on the facts provided, how should the court analyze the merits

of their First Amendment claim and with what likely outcome?

2. What are the respective burdens for each party litigating Andrew’s claim of racial

discrimination under Title VII and, based on the facts provided, what is Andrew’s

likelihood of success?

3. If A Corp files a motion to dismiss Andrew’s negligence claim, arguing that the

court should decline to exercise supplemental jurisdiction, how would the court

likely rule on the motion?

26

Page 31: JULY 2016 PENNSYLVANIA BAR EXAMINATION · JULY 2016 PENNSYLVANIA BAR EXAMINATION Essay Questions and Examiners’ Analyses and Performance Test Pennsylvania Board of Law Examiners

Question No. 4: Examiner’s Analysis

1. The court would likely find that B City’s ordinance prohibiting all leafletting in

Green Park violates the First Amendment to the United States Constitution because

it is not a reasonable time, place, and manner restriction.

The First Amendment to the United States Constitution states, in relevant part, the

following: “Congress shall make no law . . . abridging the freedom of speech.” U.S. Const.

amend. I. The First Amendment is applied to the states through the Fourteenth Amendment to

the Constitution. See Gitlow v. New York, 268 U.S. 652, 666 (1925) (assuming “freedom of

speech and of the press – which are protected by the First Amendment from abridgment by

Congress – are among the fundamental personal rights and ‘liberties’ protected by the due

process clause of the Fourteenth Amendment from impairment by the States”).

To determine whether there was a violation of First Amendment rights, the court must

first decide whether the activity is protected by the First Amendment. See Cornelius v. NAACP

Legal Def. & Educ. Fund, Inc., 473 U.S. 788, 797 (1985). If the activity is protected, the court

must identify whether the challenged restrictions impact a public or nonpublic forum, because

that determination dictates the extent to which the government can restrict the First Amendment

activities within the forum. See id. Finally, the court must determine whether the proffered

justifications for prohibiting speech in the forum satisfy the requisite standard of review. Id.

It is well established that passing out pamphlets (otherwise known as leafletting)

constitutes protected First Amendment activity. See McCullen v. Coakley, 134 S. Ct. 2518, 2536

(2014). In McCullen, the Supreme Court recently reaffirmed that leafletting and one-on-one

communications are First Amendment-protected activities, observing that in the context of

petition campaigns “one-on-one communication is the most effective, fundamental, and perhaps

economical avenue of political discourse” and “no form of speech is entitled to greater

constitutional protection” than leafletting in the advocacy of a politically controversial

viewpoint. Id. (internal quotation marks and alteration omitted) (citation omitted). The Court

explained further, “[w]hen the government makes it more difficult to engage in these modes of

communication, it imposes an especially significant First Amendment burden." Id. Thus, it is

clear that the proposed leafletting of PC is activity protected by the First Amendment.

Next, the court must identify the type of forum at issue. The Supreme Court has

traditionally identified three types of speech fora: the traditional public forum, the designated

public forum, and the nonpublic forum. See Cornelius, 473 U.S. at 802. Traditional public fora

are places that by long tradition or government fiat have been open to public assembly and

debate. Perry Educ. Ass’n v. Perry Local Educators’ Ass’n, 460 U.S. 37, 45 (1983) (“At one end

of the spectrum are streets and parks which ‘have immemorially been held in trust for the use of

the public, and, time out of mind, have been used for purposes of assembly, communicating

thoughts between citizens, and discussing public questions.’” (quoting Hague v. Comm. for

Indus. Org., 307 U.S. 496, 515 (1939)). In these traditional public fora, the government’s right

to “limit expressive activity [is] sharply circumscribed.” Id. A designated public forum is public

property, not constituting a traditional public forum, which the government has intentionally

27

Page 32: JULY 2016 PENNSYLVANIA BAR EXAMINATION · JULY 2016 PENNSYLVANIA BAR EXAMINATION Essay Questions and Examiners’ Analyses and Performance Test Pennsylvania Board of Law Examiners

“opened for use by the public as a place for expressive activity.” Id. The government is not

required to retain the open character of the property indefinitely, but “as long as it does so it is

bound by the same standards [that] apply in a traditional public forum.” Id. at 46. Nonpublic

fora is property that is not a traditional public forum and has not been designated as a public

forum. Id. “Access to a nonpublic forum . . . can be restricted as long as the restrictions are

‘reasonable and are not an effort to suppress expression merely because public officials oppose

the speaker’s view.’” Cornelius, 473 U.S. at 800 (brackets omitted) (quoting Perry Educ. Ass’n,

460 U.S. at 46). Public streets and parks are paradigm examples of the public forum. See

Pleasant Grove City, Utah v. Summum, 555 U.S. 460, 469 (2009) (citing Perry Educ. Ass’n, 460

U.S. at 45). Here, Green Park is appropriately considered a traditional public forum as the facts

provide it is a public park.

Next, the court must determine which level of scrutiny to apply to evaluate a government

restriction on speech in a public forum. In a traditional public forum, like a public park, a

“government may enforce reasonable time, place, and manner regulations as long as the

restrictions ‘are content-neutral, are narrowly tailored to serve a significant government interest,

and leave open ample alternative channels of communication.’” U.S. v. Grace, 461 U.S. 171,

177 (1983) (quoting Perry Educ. Assn., 460 U.S. at 45); see also McCullen, 134 S. Ct. at 2529

(citations omitted); McTernan v. York, 564 F.3d 636, 645 (3d Cir. 2009)(where the government

limits the time, place, or manner of speech in a traditional public forum without reference to the

subject matter of the speech or the viewpoint expressed, intermediate scrutiny applies). Here, B

City’s ordinance prohibiting all leafletting in Green Park, regardless of type, is a content-neutral

restriction on speech because the restriction covers all leafletting activity regardless of its

content.

Accordingly, because the subject ordinance is content-neutral, the court would apply

intermediate scrutiny to determine whether the regulation is a valid restraint on speech;

specifically asking whether the regulation is narrowly tailored to serve a significant government

interest and whether it leaves open ample alternative channels of communication. Here, while

the government interest in reducing litter in Green Park would likely be considered significant,

see, e.g. Members of City Council v. Taxpayers for Vincent, 466 U.S. 789, 805 (1984) (providing

“the state may legitimately exercise its police powers to advance esthetic values”), the regulation

is not narrowly tailored because it is a complete and total ban on passing out pamphlets in Green

Park and there are other methods to prevent littering such as the punishment of those who throw

paper in the park, See Schneider v. New Jersey, 308 U.S. 147, 162 (1939) (holding that the

interest of keeping the streets free of litter is insufficient to justify a law prohibiting handing

literature to a person willing to receive it; while the state can prohibit littering, it could not

prohibit the expressive activity of leafletting). Additionally, there are other ways to reduce litter

in Green Park, including fining those who are caught littering, which further indicates that the

ordinance is not narrowly tailored. Further, there are not ample alternative channels of

communication because Green Park is the only public park in B City and all leafletting is

prohibited in the park. The Supreme Court has stated, “one is not to have the exercise of his

liberty of expression in appropriate places abridged on the plea that it may be exercised in some

other place.” Schneider, 208 U.S. 163. Moreover, the Court has stated that restrictions on

leafletting impose “an especially significant First Amendment burden." McCullen, 134 S. Ct. at

28

Page 33: JULY 2016 PENNSYLVANIA BAR EXAMINATION · JULY 2016 PENNSYLVANIA BAR EXAMINATION Essay Questions and Examiners’ Analyses and Performance Test Pennsylvania Board of Law Examiners

2536. Thus, the court would likely find that the B City ordinance prohibiting leafletting of any

kind in Green Park violates the First Amendment to the United States Constitution.

2. Andrew’s claim will be analyzed under the McDonnell-Douglas burden-shifting

approach, and Andrew will likely be successful in his claim of racial discrimination.

Title VII of the Civil Rights Act of 1964, 42 U.S.C.A. § 2000e et seq., prohibits

discrimination in employment on the basis of race. Specifically, an employer may not “fail or

refuse to hire or . . . discharge any individual, or otherwise . . . discriminate against any

individual with respect to his compensation, terms, conditions, or privileges of employment,

because of such individual’s race.” 42 U.S.C.A. § 2000e-2(a)(1). An employer becomes liable

under Title VII for disparate treatment when the plaintiff “establish[es] ‘that the defendant had a

discriminatory intent or motive’ for taking a job-related action.” Ricci v. DeStefano, 557 U.S.

557, 577 (2009) (quoting Watson v. Fort Worth Bank & Trust, 487 U.S. 977, 985-86 (1988)).

When a plaintiff alleges race discrimination under Title VII, courts use the three-step

burden shifting framework developed in McDonnell Douglas Corp. v. Green, 411 U.S. 792, 802-

804 (1973), to guide the analysis of the claim. See also Texas Dep’t of Cmty. Affairs v. Burdine,

450 U.S. 248 (1981) (applying McDonnell Douglas burden shifting test to a discharge case).

The burden is first on the plaintiff to demonstrate a prima facie case of race discrimination. To

establish a prima facie case, a plaintiff must show that (1) he or she belongs to a protected class,

(2) he or she was qualified for the position, (3) he or she was subjected to an adverse

employment action, and (4) similarly situated individuals of another race were treated more

favorably. See McDonnell Douglas, 411 U.S. at 802; see also Josey v. John R. Hollingsworth,

Corp., 966 F.2d 632, 638 (3rd Cir. 1993) (applying the burden shifting test and the prima facie

elements to a discharge case). Once the plaintiff establishes a prima facie case of racial

discrimination pursuant to Title VII, the burden shifts to the defendant-employer to offer a

legitimate non-discriminatory reason for its actions. Burdine, 450 U.S. at 253, 254. If the

defendant-employer carries its “burden of production[,]” then the plaintiff has the opportunity to

demonstrate that the proffered reason is pretext, i.e. to cover up the real reason for the adverse

employment action which is discrimination. Id. at 255. When the burden shifts back to the

plaintiff, he has

the opportunity to demonstrate that the proffered reason was not the true reason

for the employment decision. This burden now merges with the ultimate burden

of persuading the court that []he has been the victim of intentional discrimination.

[][H]e may succeed in this either directly by persuading the court that a

discriminatory reason more likely motivated the employer or indirectly by

showing that the employer’s proffered explanation is unworthy of credence.

Id. at 256 (citing McDonnell Douglas, 411 U.S. at 804-05).

Here, Andrew can likely succeed on his Title VII racial discrimination claim based on

circumstantial evidence. Andrew can establish a prima facie case of race discrimination because

(1) he is an African American, thus he is a member of a protected class, (2) he was qualified for

his job because he had been hired and was employed at A Corp, there was no evidence that he

29

Page 34: JULY 2016 PENNSYLVANIA BAR EXAMINATION · JULY 2016 PENNSYLVANIA BAR EXAMINATION Essay Questions and Examiners’ Analyses and Performance Test Pennsylvania Board of Law Examiners

was not qualified, and he was considered a good employee, (3) he was terminated, an adverse

employment action, and (4) Tom, who is white and Andrew’s fellow employee, had the same

evaluations and worked on the same jobs as Andrew but was treated differently than Andrew

when he was not fired. When the burden of production shifts to A Corp to offer a legitimate,

non-discriminatory explanation for Andrew’s termination, A Corp will argue that Andrew was

fired because he missed a deadline on a big assignment, causing A Corp to lose a major client, C

Corp. This will likely satisfy A Corp’s burden of production of offering a legitimate, non-

discriminatory reason for Andrew’s discharge. However, Andrew will likely be able to prove

that A Corp’s proffered reason is pretext because Andrew was A Corp’s only African American

employee, Andrew and Tom’s boss did not fire Tom, even though Tom also missed the same big

deadline, both Andrew and Tom received the same above average evaluation, and both had the

same boss. Further, Andrew and Tom’s boss’s statement that “[w]e don’t need people like you

working here” can be construed to mean that the boss fired Andrew because of his race. Finally,

during discovery it was revealed that Andrew’s boss had sent emails from his work email

account containing racist jokes to friends and colleagues within the last six months of Andrew’s

employment. Based on this combination of facts, it would appear that Andrew would meet his

burden that A Corp’s asserted justification is a pretext for discrimination. Accordingly, it is

likely that a court would find that Andrew has met his burden to show that he was discriminated

against on the basis of his race by A Corp under Title VII of the Civil Rights Act and would be

successful on his claim.

It is also possible that the court could analyze Andrew’s claim under a “mixed-motive”

framework. Title VII was amended in 1991 to provide, in relevant part, “an unlawful

employment practice is established when the complaining party demonstrates that race . . . was a

motivating factor for any employment practice, even though other factors also motivated that

practice.” 42 U.S.C.A. § 2000e-2; see Desert Palace, Inc. v. Costa, 539 U.S. 90, 94 (2003)

(stating that Congress passed the 1991 amendment to address standards to prove discrimination

in a mixed motive case has occurred and to provide employers with a limited affirmative

defense). In a mixed motive action, a plaintiff must prove by direct or circumstantial evidence

that an illegitimate factor had a “motivating” role in the employment decision. Price

Waterhouse v. Hopkins, 490 U.S. 228 (1989), Desert Palace, 539 U.S. at 94. If the plaintiff

succeeds, the employer may limit liability by proving that it would have made the same decision

even in the absence of the impermissible factor. 42 U.S.C.A. § 2000e-5(g)(2)(B); see Desert

Palace, 539 U.S. at 94-95. Here, Andrew has both direct and circumstantial evidence of

illegitimate factors (i.e., the statement of his boss that “we don’t need people like you working

here, Andrew” as well as the racist emails sent by his boss). Since we know that Tom was not

terminated for the same conduct as Andrew, there is likely sufficient evidence of a

discriminatory motive for Andrew to prevail under a mixed-motive analysis. A Corp could limit

its liability, however, by proving that it would have made the same decision to terminate Andrew

even in the absence of the impermissible factors. Under the facts presented in this question, it

would be extremely difficult for A Corp to avoid liability.

30

Page 35: JULY 2016 PENNSYLVANIA BAR EXAMINATION · JULY 2016 PENNSYLVANIA BAR EXAMINATION Essay Questions and Examiners’ Analyses and Performance Test Pennsylvania Board of Law Examiners

3. The court will likely grant A Corp’s motion to dismiss the negligence claim because

the claims do not arise out of the same case or controversy.

28 U.S.C.A. § 1331 provides that a “district court[] shall have original jurisdiction of all

civil actions arising under the Constitution, laws, or treaties of the United States.” 28 U.S.C.A. §

1331. Likewise, Title VII confers jurisdiction to federal district courts to hear discrimination

claims. 42 U.S.C.A. § 2000e-5(f)(3). Once it is established that a district court has original

jurisdiction to hear a federal claim, i.e. a claim under Title VII, the court may consider whether

to also hear related state law claims. See 28 U.S.C.A. § 1367(a).

Regarding supplemental jurisdiction, in 1990, Congress codified decades of

jurisprudence to statutorily provide the district courts with supplemental jurisdiction over, inter

alia, certain claims that are “part of the same case or controversy” as other claims over which the

district courts have original jurisdiction. 28 U.S.C.A. § 1367, added by Act of Dec. 1, 1990,

Pub. L. 101-650, Title III, § 310(a), 104 Stat. 5113; see Chicago v. Int’l Coll. of Surgeons, 522

U.S. 156, 165 (1997) (explaining that Congress codified the existing jurisprudence and principles

pertaining to pendent and ancillary jurisdiction in 28 U.S.C.A. § 1367). Pursuant to this statute,

if a federal court has original jurisdiction over a case because of a federal question, it also has

“supplemental jurisdiction over all other claims that are so related to claims in the action within

such original jurisdiction that they form part of the same case or controversy under Article III of

the United States Constitution.” 28 U.S.C.A. § 1367(a). The “other” claim (i.e. the claim over

which the district court does not have original jurisdiction) can be referred to as a “pendent”

claim. See Int’l College of Surgeons, 522 U.S. at 164-65.

To determine whether the original claim and the potentially pendent claim are part of the

“same case or controversy,” a federal court queries whether the pendent claim arises out of a

“common nucleus of operative fact[s]” with the properly asserted federal claim, such that “the

relationship between the [federal] claim and the state claim permits the conclusion that the entire

action before the court comprises but one constitutional ‘case’” and a plaintiff “would ordinarily

be expected to try them all in one judicial proceeding.” United Mine Workers of Am. v. Gibbs,

383 U.S. 715, 725, (1966) (footnote omitted); see also Int’l College of Surgeons, 522 U.S. at

165-66 (explaining that Congress codified the jurisprudence and principles of Gibbs pertaining to

pendent jurisdiction in 28 U.S.C.A. § 1367). If a court determines that it has the power to hear a

pendent claim because it arises out of a common nucleus of operative fact, it is still within the

court’s discretion whether to exercise supplemental jurisdiction over the claim. 28 U.S.C.A. §

1367(c); see also Gibbs, 383 U.S. at 726; Int’l College of Surgeons, 522 U.S. at 165.

Here the district court will likely dismiss the negligence claim because it is not part of the

same case or controversy as the Title VII racial discrimination claim. Andrew’s federal claim is

based on the district court’s original jurisdiction over the Title VII racial discrimination claim.

See 42 U.S.C.A. § 2000e-5(f)(3). Although the incident giving rise to the negligence claim

occurred on A Corp’s property, it cannot be said that the two claims arise out of a common

nucleus of operative fact. The racial discrimination claim concerns the motives of A Corp in

terminating Andrew while the negligence claim concerns whether A Corp breached a duty to

Andrew in failing to remove ice in its parking lot. The claims appear to be too attenuated to be

31

Page 36: JULY 2016 PENNSYLVANIA BAR EXAMINATION · JULY 2016 PENNSYLVANIA BAR EXAMINATION Essay Questions and Examiners’ Analyses and Performance Test Pennsylvania Board of Law Examiners

joined in the same federal case. Thus, the court will likely dismiss Andrew’s negligence claim,

and Andrew can file the negligence claim in state court.

It must be noted that the facts here do not implicate or suggest that diversity jurisdiction

would be available to bring the state law negligence claim into federal court. Pursuant to 28

U.S.C. § 1332, a federal court may have original jurisdiction over a civil claim where the amount

in controversy exceeds the sum or value of $75,000 and is between citizens of different States.

However, there are no facts in the question to suggest that the negligence claim could be subject

to diversity jurisdiction.

32

Page 37: JULY 2016 PENNSYLVANIA BAR EXAMINATION · JULY 2016 PENNSYLVANIA BAR EXAMINATION Essay Questions and Examiners’ Analyses and Performance Test Pennsylvania Board of Law Examiners

Question No. 4: Grading Guidelines

1. Constitutional Law

Comments: Applicants should demonstrate an understanding that a public park is considered a

public forum for purposes of a First Amendment freedom of speech analysis and should

recognize that a blanket prohibition on leafletting/pamphleteering in a public park violates First

Amendment protections.

9 points

2. Title VII Racial Discrimination

Comments: Applicants should demonstrate an understanding of the requirements for making a

disparate treatment claim under Title VII of the Civil Rights Act of 1964 for racial

discrimination, specifically the McDonnell-Douglas burden-shifting analysis applied to such

claims, and should apply the law to the facts to reach a well-reasoned conclusion.

6 points

3. Civil Procedure

Comments: Applicants should demonstrate knowledge of the requirements for supplemental

jurisdiction, and apply the law to the facts to determine that Andrew’s Title VII racial

discrimination claim and his common law negligence claim do not form the “same case or

controversy” as anticipated by the supplemental jurisdiction statute.

5 points

33

Page 38: JULY 2016 PENNSYLVANIA BAR EXAMINATION · JULY 2016 PENNSYLVANIA BAR EXAMINATION Essay Questions and Examiners’ Analyses and Performance Test Pennsylvania Board of Law Examiners

Question No. 5

Ted, a wealthy businessman, conveyed Blackacre, a very valuable tract of land in Big

City, Pennsylvania, by a validly recorded deed to himself and his son, Don, as joint tenants with

right of survivorship. Ted also signed and delivered a valid deed to Whiteacre, his vacation

property adjacent to the Snowtop Ski Resort in Rural County, Pennsylvania, to his daughter,

Carly. Carly accepted the deed but never recorded it.

Ted always wanted Don to pursue a career in politics. At his father’s urging, Don

announced his candidacy for his party’s nomination for U.S. Senator from Pennsylvania. Don

contacted Karl, who previously handled public relations for Ted’s business, about designing

brochures to be mailed to “supervoters,” producing radio and television ads for Don’s campaign,

and arranging for the purchase of air time in major media markets throughout the state to run the

ads. After extensive negotiations about the fees for Karl’s performance of these services for

Don’s campaign and careful review of a final draft agreement, Don and Karl signed a “Media

Contract.” The Media Contract explicitly stated that Don would pay Karl a flat fee for each

brochure mailed to a supervoter and a 5% commission based on the cost charged by each radio

and television station to broadcast the ad at a particular time. The Media Contract further stated,

“This writing is intended as the final, complete and exclusive expression of the terms of our

agreement.” To pay for some of the costs under the Media Contract, Don executed a mortgage

on his interest in Blackacre.

After a hard day working on Don’s campaign, Karl went to the Dew Drop Inn (the “Inn”)

to relax and have a beer. That evening, the Inn was holding a publicly advertised “bar trivia

contest.” The terms of the contest, which had no formal entry requirements, publicly stated that

the Inn would give free beer for life to the person in attendance who had the most correct

34

Page 39: JULY 2016 PENNSYLVANIA BAR EXAMINATION · JULY 2016 PENNSYLVANIA BAR EXAMINATION Essay Questions and Examiners’ Analyses and Performance Test Pennsylvania Board of Law Examiners

answers to trivia questions posted via a tablet connected to an interactive electronic scoreboard.

Karl, who had no knowledge of the contest, started to answer the questions on a tablet, telling the

Inn’s manager, “I’m just playing for fun to take my mind off things.” As the final contest

question was about to be posted, Karl’s waitress exclaimed to him, “If you get this right, you’ll

win!” Quickly learning about the contest offer details from the waitress, Karl decided to try to

win the contest and prize. After Karl correctly answered the final question and attempted to

claim the prize, the Inn’s manager refused to declare Karl the winner of the contest saying that

Karl was not really participating in the contest but was simply “playing for fun.”

Before the election, Carly told a reporter that she was voting for Don’s opponent because

Don was too beholden to special interests. Furious at Carly for failing to support her brother’s

candidacy, Ted conveyed Whiteacre to Don by a properly executed deed for “mutual love and

affection.” Don, who had no knowledge of Ted’s prior deed giving Whiteacre to Carly,

immediately recorded his deed.

1. Ted unexpectedly died after the election. By will, Ted left his entire estate to the

Big City Charitable Trust. Who owns Blackacre following Ted’s death?

2. After learning about Carly’s deed, Don filed a quiet title action in Rural County

claiming that he is the owner of Whiteacre. Will Don’s action be successful?

3. When the Inn refused to give him free beer for life, Karl sued the Inn for breach

of contract. Will Karl prevail in his suit?

4. When Don only paid a 3% commission for the broadcasting of the radio and

television campaign ads, Karl sued Don for breach of the Media Contract. At

trial, Don attempted to introduce evidence that he and Karl orally agreed just prior

to the signing of the Media Contract that Karl would be paid a 3% commission,

the same rate that Karl charged for public relations work for Ted’s business. If

Karl objects to this evidence based on the parol evidence rule, how should the

court rule?

35

Page 40: JULY 2016 PENNSYLVANIA BAR EXAMINATION · JULY 2016 PENNSYLVANIA BAR EXAMINATION Essay Questions and Examiners’ Analyses and Performance Test Pennsylvania Board of Law Examiners

Question No. 5: Examiner’s Analysis

1. Because the joint tenancy with right of survivorship was severed by Don’s

mortgage, Don and the Big City Charitable Trust own Blackacre as tenants in

common following Ted’s death.

In Lafayette v. Brinham, 363 Pa. 360, 69 A.2d 130 (1949), the Pennsylvania Supreme

Court held that the Uniform Interparty Act of 1927, 69 P.S. § 541 et seq., allowed a grantor to

create a valid joint tenancy with right of survivorship by directly conveying the property to the

grantor and another party without first having to convey the property to a straw party.

Therefore, when Ted directly conveyed Blackacre to Don and himself, he created a valid joint

tenancy with right of survivorship under Pennsylvania law.

A joint tenancy with right of survivorship is created by the co-existence of the four

unities of interest, title, time and possession. In Re Estate of Quick, 588 Pa. 485, 490, 905 A.2d

471, 474 (2006). The essence of this form of concurrent ownership is to vest title equally in two

or more persons during their lifetimes, with sole ownership passing to the survivor upon the

death of the other joint tenant or tenants. In Re Parkhurst’s Estate, 402 Pa. 527, 532, 167 A.2d

476, 478 (1961). “The survivorship characteristic of a joint tenancy precludes a joint tenant from

disposing of his interest by will.” Nicholson v. Johnston, 855 A.2d 97, 100 (Pa. Super. 2004)

(quoting General Credit Co. v. Cleck, 415 Pa.Super. 338, 609 A.2d 553, 556 (1992)) (further

citation omitted).

A joint tenancy with right of survivorship is severable by the action, voluntary or

involuntary, of any of the joint tenants that destroys one of the required four unities. “It is . . .

well settled that a joint tenancy in real estate with the right of survivorship is severable by the

act, voluntary or involuntary, of either of the parties.” Sheridan v. Lucey, 395 Pa. 306, 308, 149

A.2d 444, 445 (1959). Upon the occurrence of an act of severance, the joint tenancy with right

of survivorship becomes a tenancy in common. Stanger v. Epler, 382 Pa. 411, 417, 115 A.2d

197, 200 (1955). Each tenant in a tenancy in common possesses the power to dispose of his

interest or any portion thereof by deed or by will. Boyer, Ralph E., Hovenkamp, Herbert, &

Kurtz, Sheldon F., The Law of Property: An Introductory Survey, § 5.3, at p. 116 (West

Publishing Co. 4th ed.).

Jurisdictions are divided concerning the effect of one joint tenant’s execution of a

mortgage upon a joint tenancy with right of survivorship. In states adhering to the “title theory”

of mortgages, one joint tenant’s execution of a mortgage is viewed as a transfer of title resulting

in the severance of the joint tenancy. In states adhering to the “lien theory” of mortgages, one

joint tenant’s execution of a mortgage is viewed as mere security for repayment and is not a

transfer of title resulting in the severance of the joint tenancy. See generally, 4 Thompson on

Real Property, § 31.08 (b), at 49-50 (1994). Although Pennsylvania subscribes to the lien theory

of mortgages, see, In Re City of Philadelphia, 360 Pa. 589, 592, 63 A.2d 42, 43 (1949), whether

the execution of a mortgage operates to completely sever a joint tenancy with right of

survivorship and create a tenancy in common under Pennsylvania law is dependent upon how

many joint tenants execute the mortgage. “Where fewer than all of the joint tenants execute[] a

36

Page 41: JULY 2016 PENNSYLVANIA BAR EXAMINATION · JULY 2016 PENNSYLVANIA BAR EXAMINATION Essay Questions and Examiners’ Analyses and Performance Test Pennsylvania Board of Law Examiners

mortgage on the joint tenancy property, the mortgage . . . effectuate[s] a severance of the joint

tenancy” with right of survivorship between the joint tenant whose actions caused the severance

and the other co-tenant or tenants. General Credit Co. v. Cleck, 415 Pa. Super. 338, 346 & n.2,

609 A.2d 553, 557 & n.2 (1992), appeal discontinued, 531 Pa. 655, 613 A.2d 560 (1992) (citing,

Simpson v. Ammons, 1 Binn. 175 (Pa. 1806)). The resulting interest of the tenant who caused the

severance becomes as a tenant in common in relation to the other remaining tenant or tenants.

American Oil Co. v. Falconer, 136 Pa. Super. 598, 605, 8 A.2d 418, 421 (1939). Where all of

the joint tenants execute the mortgage on the joint tenancy property, the mortgage does not

effectuate a severance of the joint tenancy with right of survivorship because the unities

necessary for a joint tenancy with right of survivorship are considered to be preserved. General

Credit Co., 415 Pa. Super. at 346, 609 A.2d at 557 (citing Estate of Kotz, 486 Pa. 444, 458, 406

A.2d 524, 532 (1979)).

The facts state that Ted and Don owned Blackacre as joint tenants with right of

survivorship. The facts further state that only Don executed the mortgage on Blackacre. By

executing the mortgage, Don caused a severance of the joint tenancy with right of survivorship

resulting under Pennsylvania law in Ted and Don each owning an undivided one-half interest in

Blackacre as tenants in common. When Ted died, his undivided one-half interest as a tenant in

common in Blackacre passed by will to the Big City Charitable Trust. Therefore, Don and the

Big City Charitable Trust are the owners of Blackacre as tenants in common.

2. Don’s action to quiet title will not be successful because Carly has title to Whiteacre,

even though she did not record her deed, as Don is not a bona fide purchaser for

value without notice under Pennsylvania’s recording statute.

Delivery of an executed deed is all that is necessary to pass title between a grantor and a

grantee. Malamed v. Sedelsky, 367 Pa. 353, 358, 80 A.2d 853, 856 (1951). Recording of a deed

is not essential to establish the passage of title. In re Pentrack’s Estate, 486 Pa. 237, 240, 405

A.2d 879, 880 (1979).

Moreover, failure to record a deed does not render it void under all circumstances or as to

all persons. An unrecorded deed gives the grantee an equitable interest in the property and is

valid against the grantor and the grantor’s heirs and devisees. It also is valid against certain

subsequent grantees who may claim ownership of the property. Land v. Commonwealth of

Pennsylvania, 101 Pa. Cmwlth. 179, 183-84, 515 A.2d 1024, 1025-26 (1986) (interpreting 21

P.S. § 351); 2 Ronald M. Friedman, Ladner Pennsylvania Real Estate Law, § 19.03 (George T.

Bisel Co., Inc. 6th ed. 2015).

Pennsylvania statutory law attempts to eliminate the possibility of fraud resulting from

the failure to record a deed. Friedman, supra at § 19.04. Thus, Pennsylvania’s recording statute

gives a subsequent grantee priority over the equitable estate of the first grantee. Long John

Silver’s, Inc. v. Fiore, 255 Pa. Super. 183, 190, 386 A.2d 569, 572-73 (1978) (citation omitted).

To attain the priority and the protection afforded by the recording statute, the subsequent grantee

must: (1) be a bona fide purchaser for value of the property; (2) be without actual or constructive

notice of the equitable interest of the prior grantee; and (3) record a deed to the property before

the first grantee records. 21 P.S. § 351; Fiore, 255 Pa. Super. at 190, 386 A.2d at 572-73 (citing,

37

Page 42: JULY 2016 PENNSYLVANIA BAR EXAMINATION · JULY 2016 PENNSYLVANIA BAR EXAMINATION Essay Questions and Examiners’ Analyses and Performance Test Pennsylvania Board of Law Examiners

Lund v. Heinrich, 410 Pa. 341, 189 A.2d 581 (1963); Overly v. Hixson, 169 Pa. Super. 187, 82

A.2d 573 (1951)). “A bona fide purchaser [for value] is defined as one who pays valuable

consideration, has no notice of outstanding rights of others, and acts in good faith.” Carnegie

Nat. Gas Co. v. Braddock, 142 Pa. Cmwlth. 383, 387, 597 A.2d 285, 288 (1991) (citation

omitted).

The facts state that Don recorded his deed to Whiteacre and that Carly never recorded her

deed. The facts further state that Don had no knowledge of Ted’s prior deed giving Whiteacre to

Carly. But Don was not a bona fide purchaser for value because he did not give anything of

value in exchange for Ted’s conveyance of Whiteacre. Rather, he received Whiteacre as a donee

because the consideration recited in Ted’s deed to Don was “mutual love and affection.”

Although it was not recorded, Ted’s earlier deed passed title to Whiteacre to Carly.

Because Don was not a bona fide purchaser for value, he will not receive the protection of

Pennsylvania’s recording statute. Therefore, Don’s quiet title action will not be successful.

3. Karl will be successful in his breach of contract action against the Inn because Karl

accepted the Inn’s offer to award a prize by rendering the requested performance.

The essential elements to the formation of a contract are an offer, an acceptance, and

consideration. Schreiber v. Olan Mills, 426 Pa. Super. 537, 541, 627 A.2d 806, 808 (1993)

(citations omitted).

“An offer is the manifestation of willingness to enter into a bargain [which would] justify

another person in understanding that his assent to that bargain is invited and will conclude it.”

Restatement (Second) of Contracts, § 24 (Am. Law Inst. 1981). “An offer creates a power of

acceptance in a[n] . . . offeree to transform the offeror’s promise into a contractual obligation.”

Philadelphia Newspapers, Inc. v. Unemployment Comp. Bd. of Review, 57 Pa. Cmwlth. 639, 641

n.3, 426 A.2d 1289, 1290 n.3 (1981) (citing Restatement (Second) of Contracts, § 24).

Consideration is an “essential element of an enforceable contract.” Stelmack v. Glen Alden Coal

Co., 339 Pa. 410, 413, 14 A.2d 127, 128 (1940). “Valid ‘consideration’ confers a benefit upon

the promisor or causes a detriment to the promissee and must be an act, forbearance or return

promise bargained for and given in exchange for the original promise.” Cardamone v. Univ. of

Pittsburgh, 253 Pa. Super. 65, 72 n.6, 384 A.2d 1228, 1232 n.6 (1978) (citation omitted).

Acceptance of an offer occurs when the offeree assents to the terms “in a manner invited

or required by the offer.” Schott v. Westinghouse Elec. Corp., 436 Pa. 279, 288, 259 A.2d 443,

447 (1969) (citation omitted). An offer can be accepted by performance of an act when the offer

invites such an acceptance. Restatement (Second) of Contracts, §§ 30(1), 53(1). Pennsylvania

law follows the general rule that a contest promoter’s declaration of the conditions and rules for

awarding a prize constitutes a valid offer to enter into a contract. Cobaugh v. Klick-Lewis, Inc.,

385 Pa. Super. 587, 590, 561 A.2d 1248, 1249 (1989). An offer which invites an offeree to

accept by rendering a performance and does not invite a promissory acceptance is an “offer for a

unilateral contract.” Restatement (Second) of Contracts, § 45 & cmt. a; Martin v. Capital Cities

Media, Inc., 354 Pa. Super. 199, 211, 511 A.2d 830, 836 (1986) (stating “[a] unilateral contract

is a contract wherein one party makes a promissory offer which calls for the other party to accept

38

Page 43: JULY 2016 PENNSYLVANIA BAR EXAMINATION · JULY 2016 PENNSYLVANIA BAR EXAMINATION Essay Questions and Examiners’ Analyses and Performance Test Pennsylvania Board of Law Examiners

by rendering a performance”). Offers of rewards and prizes where the only acceptance that is

necessary is the performance of the act requested are typical cases in which a unilateral contract

is offered. Restatement (Second) of Contracts, § 45, cmt. a.

An offeree’s performance constitutes an acceptance only if he knows of the offer. Id. at §

51, cmt. a. If the act requested in the offer is performed by a person who has knowledge of the

offer, the presumption is very strong that he or she acted with reference to the offer though

perhaps having had one or more superior motivations for performance of the requested act.

Absent a manifestation of an intention not to accept the offer, the performance evidences

acceptance of the offer. John E. Murray, Jr., Murray on Contracts, § 45 [C], at 145-146 (5th ed.

2011); see also, Restatement (Second) of Contracts, § 53(3).

An act begun without knowledge of the offer but which is completed after the offeree

gains knowledge of the offer may constitute an acceptance. Restatement (Second) of Contracts, §

51. The Restatement (Second) of Contracts takes this position based upon the view “that

knowledge of the offer [gained] after part performance can induce the offeree to complete [the]

performance and, since part performance is valueless to the offeror, there is a reasonable

inference that the offeror intends to create a power of acceptance in a party who has yet to

complete the performance required by the offer. Murray on Contracts, supra, § 45, at 144;

Restatement (Second) of Contracts, § 51, cmt. b.

Furthermore, the existence of mixed motives to perform a requested act does not preclude

a finding of acceptance. Murray on Contracts, supra, § 44 C. The offeree may have many or

different reasons or motives for creating or exercising a power of acceptance and the motivation

to enter into a contract may be subsidiary to other motivations. As long as the offeree has

knowledge of the offer, it is not necessary that the sole motivation of the offeree has to be his

desire for the offered reward. Murray on Contracts, supra, § 44 C.

In this case, the performance required to accept the Inn’s offer was to be the person in

attendance who had the most correct answers to trivia questions posted via a tablet to an

interactive electronic scoreboard. Therefore, the Inn’s offer required an acceptance by

performance.

Although Karl was not aware of the offer at the time that he started performance, he

became aware of the offer before he answered the final trivia question and obtained the highest

score. Thus, Karl rendered the requested performance with knowledge of the offer. Moreover,

while Karl initially stated that he was answering the trivia questions “for fun to take my mind off

things,” he changed his mind after obtaining knowledge of the offer. There is nothing which

rebuts the presumption that his completed performance was an acceptance of the offer. At best,

the facts show that Karl’s performance of the requested act was rendered with mixed motives.

In conclusion, Karl had knowledge of the offer and accepted the offer by rendering the

requested performance thereby creating a unilateral contract between the parties. Therefore,

Karl’s breach of contract action against the Inn should be successful.

39

Page 44: JULY 2016 PENNSYLVANIA BAR EXAMINATION · JULY 2016 PENNSYLVANIA BAR EXAMINATION Essay Questions and Examiners’ Analyses and Performance Test Pennsylvania Board of Law Examiners

4. The court likely would sustain Karl’s parol evidence rule objection to Don’s attempt

to introduce evidence that the parties orally agreed prior to signing the Media

Contract that Karl would receive a 3% commission.

The purpose of the parol evidence rule is to “preserve the integrity of written agreements

by refusing to permit the contracting parties to attempt to alter the import of their contract

through the use of [prior or] contemporaneous oral declarations.” Rose v. Food Fair Stores, Inc.,

437 Pa. 117, 120-21, 262 A.2d 851, 853 (1970); Yocca v. Pittsburgh Steelers Sports, Inc., 578

Pa. 479, 497, 854 A.2d 425, 436 (2004) (further citation omitted). The Pennsylvania Supreme

Court has explained the parol evidence rule as follows:

Where the parties, without any fraud or mistake, have deliberately put their

engagements in writing, the law declares the writing to be not only the best, but

the only, evidence of their agreement. All preliminary negotiations, conversations

and verbal agreements are merged in and superseded by the subsequent written

contract . . . [.] [U]nless fraud, accident or mistake be averred, the writing

constitutes the agreement between the parties, and its terms and agreements

cannot be added to nor subtracted from by parol evidence.

Yocca, 578 Pa. at 497, 854 A.2d at 436 (quoting, Gianni v. Russell & Co., 281 Pa. 320, 323, 126

A. 791, 792 (1924)).

Whether parol evidence is admissible to vary the terms of a written agreement involves a

two-fold determination. First, a court must determine whether the written agreement represents

the entire contract between the parties. Green Valley Dry Cleaners, Inc. v. Westmoreland Cty.

Indus. Dev. Corp., 832 A.2d 1143, 1154 (Pa. Cmwlth. 2003), appeal denied, 578 Pa. 696, 851

A.2d 143 (2004). If the written document “appears to be a contract complete within itself

‘couched in such terms as import a complete legal obligation without any uncertainty as to the

object or [the] extent of the engagement, it is conclusively presumed that the whole engagement

of the parties, and the extent and manner of their undertaking, were reduced to writing[.]’”

Gianni, 281 Pa. at 323, 126 A. at 792 (quoting, Seitz v. Brewers’ Refrigerating Mach. Co., 141

U.S. 510, 517 (1891)). The presence of an integration clause in a contract is viewed as a clear

and persuasive sign that the writing contains the entire agreement and expresses all of the

parties’ “negotiations, conversations, and agreements made prior to its execution[.]” McGuire v.

Schneider, Inc., 368 Pa. Super. 344, 349, 534 A.2d 115, 117 (1987), aff’d per curiam, 519 Pa.

439, 548 A.2d 1223 (1988).

Here, the facts state that after extensive negotiations about Karl’s fees for his services to

Don’s campaign and a careful review of a final draft agreement, Don and Karl signed a contract

explicitly stating that Karl would receive a 5% commission for his media services for the

campaign based on the cost charged by each radio and television station to broadcast the ad at a

particular time. Further, the written contract contained an integration clause which stated, “This

writing is intended as the final, complete and exclusive expression of the terms of our

agreement.” Based upon these facts, a court likely would conclude that the written contract

between Karl and Don constituted the entire agreement between the parties.

40

Page 45: JULY 2016 PENNSYLVANIA BAR EXAMINATION · JULY 2016 PENNSYLVANIA BAR EXAMINATION Essay Questions and Examiners’ Analyses and Performance Test Pennsylvania Board of Law Examiners

Second, a court must examine whether the prior oral agreement or statement comes

within the field embraced by the subsequent written agreement. Green Valley, 832 A.2d at 1155.

This is done by comparing the oral and written terms and determining whether the parties would

naturally and normally have included the oral term in the written agreement. “If [the terms]

relate to the same subject-matter and are so interrelated that both would be executed at the same

time, and in the same contract, the scope of the subsidiary agreement must be taken to be

covered by the writing.” Gianni, 281 Pa. at 323-24, 126 A. at 792. Since the prior oral

agreement and written agreement both relate to the same subject, i.e. the amount of the

commission that Karl would receive for performing media services for Don’s campaign, it is

reasonable to assume that the subject of the commission for the preparation and broadcasting of

the campaign ads discussed by the parties would naturally and normally be covered by the

subsequent written agreement.

Because the prior oral agreement that Don seeks to introduce directly contradicts the

integrated written contract, the court likely would sustain Karl’s objection based upon the parol

evidence rule. Thus, Don’s attempt to admit evidence that he and Karl orally agreed prior to

signing the Media Contract that Karl would receive a 3% commission instead of the 5%

commission stated in the Media Contract would likely be unsuccessful.

41

Page 46: JULY 2016 PENNSYLVANIA BAR EXAMINATION · JULY 2016 PENNSYLVANIA BAR EXAMINATION Essay Questions and Examiners’ Analyses and Performance Test Pennsylvania Board of Law Examiners

Question No. 5: Grading Guidelines

1. Effect of a Mortgage on Property Owned as Joint Tenants with Right of

Survivorship

Comments: Candidates should recognize the essential characteristics of the form of concurrent

ownership known as joint tenants with right of survivorship. Based upon the stated facts,

candidates should apply the principles of Pennsylvania law regarding how a mortgage affects

this form of ownership in reaching a well-reasoned conclusion regarding who is the owner of the

property.

5 Points

2. Recording Statute; Protection of a Bona Fide Purchaser for Value

Comments: Candidates should recognize that an unrecorded deed to real property can pass valid

title to a grantee. Candidates should discuss how Pennsylvania law encourages the recording of

deeds by giving priority to a subsequent grantee over such prior grantee who has not recorded a

deed if the subsequent grantee is a bona fide purchaser for value without notice of the prior deed

and records first. Candidates should apply these principles and reach the conclusion that the

subsequent grantee is not afforded the protection of Pennsylvania’s recording statute.

5 points

3. Contract Formation – Offer of Prize or Rewards - Acceptance by Performance

Comments: Candidates should discuss the basics of contract formation in the context of prizes

and rewards. Candidates should recognize that offers of rewards and prizes are typical cases in

which a unilateral contract is formed. Candidates also should recognize that an act begun

without knowledge of the offer, but that is completed after the offeree gains knowledge of the

offer, may constitute an acceptance and that an offeree’s mixed motives in performing a

requested act does not preclude a finding of acceptance. Candidates should apply these

principles to the stated facts in reaching a conclusion regarding the merits of the alleged breach

of contract claim.

5 Points

4. Parol Evidence Rule

Comments: Candidates should discuss the requirements of the parol evidence rule and should

apply the requirements of the rule in reaching a well-reasoned conclusion concerning whether

the rule would bar the admission of extrinsic evidence which would vary the terms of a written

agreement.

5 Points

42

Page 47: JULY 2016 PENNSYLVANIA BAR EXAMINATION · JULY 2016 PENNSYLVANIA BAR EXAMINATION Essay Questions and Examiners’ Analyses and Performance Test Pennsylvania Board of Law Examiners

Question No. 6

Kilns, Inc. (“KI”) is a Pennsylvania corporation that manufactures and sells hand crafted clay

pottery. KI has always had and continues to have the same three shareholders: Al, Ben, and Carl,

who each own one-third of KI’s shares. Al, Ben, and Al’s son are KI’s sole directors and officers

(Al is president, Ben is secretary, and Al’s son is treasurer). Carl resigned a seat on the board three

years ago when he moved out of state.

Able has served as KI’s attorney for the past ten years. He does not and never has

represented any of KI’s shareholders or directors personally. Last month, Able discovered, from a

review of internal corporate records, that the current KI directors had made a secret deal with a new

supplier of raw materials to KI (“Supplier”). Under the deal, Supplier would increase the price it

charged KI for materials and would then pay KI’s directors a monthly “finder’s fee” consisting of

50% of Supplier’s profit on the materials for each month. When this contract with Supplier was

presented at a board meeting, it was summarily approved. The Supplier deal has negatively affected

KI’s profits and will significantly reduce future profits and distributions to KI’s shareholders.

After learning of the board’s secret deal with Supplier, Able confronted Al and correctly

advised him that the board was acting against KI’s interests in breach of its fiduciary duty to KI.

Incensed by Able’s comments, Al told Able, “you work for me so you better keep your mouth shut.”

Al further threatened to fire Able as corporate counsel and to sue him if he discussed the deal with

anyone, including the other board members or Carl.

Two weeks ago, Al, who routinely orders packaging material and boxes to ship KI’s goods,

telephoned Boxes, Inc. (“Boxes”), a supplier of packaging materials and boxes, and, on behalf of KI,

ordered 500 padded boxes at a total cost of $2,000. The day after the call, Boxes faxed a confirming

memo to Al accepting the telephone order. The confirming memo included the following provision,

43

Page 48: JULY 2016 PENNSYLVANIA BAR EXAMINATION · JULY 2016 PENNSYLVANIA BAR EXAMINATION Essay Questions and Examiners’ Analyses and Performance Test Pennsylvania Board of Law Examiners

which was not discussed by Al or Boxes when the order was placed: “Any sums outstanding on this

order for more than 30 days shall bear interest at a rate of 1.5% per month (18% annually).” Al read

the confirming memo and filed it away without taking any further action.

Last week, Betty visited KI’s showroom looking to buy a pottery bowl. She explained to

KI’s salesman that she wanted to use the bowl to heat food in her microwave. The salesman showed

her several bowls and guaranteed her that any of the bowls would be suitable and safe for use in a

microwave. Betty bought one of the bowls. Yesterday, Betty was having a party at her house and

asked her sister, who was a guest at the party, to use Betty’s microwave oven to heat some nacho

cheese dip using the bowl that Betty had purchased from KI. Betty’s sister placed the bowl in the

microwave, which was working properly, and was standing in front of it waiting for the dip to warm

when the bowl suddenly burst, causing the glass door of the microwave to shatter. The shattered

glass and hot dip hit Betty’s sister in the face causing severe cuts and burns.

1. Under the Pennsylvania Rules of Professional Conduct, what responsibilities does

Able have and what steps should he take relative to the board and Carl with respect to

the Supplier contract and secret deal?

2. Assume Carl has learned about the Supplier contract which is still in effect, and Carl

is outraged. On behalf of KI, what type of action could be filed by Carl to recover

lost profits as a result of the deal, and what procedural steps would need to be taken

by Carl relative to the board of directors of KI prior to filing the action?

3. Assume the confirming memo from Boxes satisfies any statute of frauds requirement

under the Uniform Commercial Code (the “Code”). Will Boxes be able to enforce

the interest provision in its confirming memo if KI does not pay within 30 days?

4. Assume that all implied warranties that might apply to the sale of the bowl have been

properly disclaimed. Given the failure of the bowl, can Betty’s sister assert a claim

under the Code against KI?

44

Page 49: JULY 2016 PENNSYLVANIA BAR EXAMINATION · JULY 2016 PENNSYLVANIA BAR EXAMINATION Essay Questions and Examiners’ Analyses and Performance Test Pennsylvania Board of Law Examiners

Question No. 6: Examiner’s Analysis

1. Able represents KI and should discuss the Supplier deal with the board and

recommend that they reconsider the deal. If the board rejects Able’s advice, Able

could disclose his knowledge of the deal made by the board to Carl in order to

protect the interests of KI.

The facts implicate multiple issues under the Pennsylvania Rules of Professional Conduct

(the “RPC”) that Able would need to refer to and analyze in determining how to proceed. Able

is clearly faced with a situation where the actions of the KI board are in breach of their fiduciary

duties to the corporation because the actions are materially harming the corporation and its

shareholders.

Able must first define who his client is. RPC 1.13(a) provides, “[a] lawyer employed or

retained by an organization represents the organization acting through its duly authorized

constituents.” Pa.R.P.C. No. 1.13(a). Able represents KI and does not, without more (e.g.,

consent to joint or multiple representation under Pa.R.P.C. No. 1.13(e)), become the lawyer for

any of the entity’s officers, directors, or shareholders. Thus, Able’s client is the corporation and

his first and foremost duty is to KI and not to KI’s board members individually. RPC 1.13(d)

further provides that, “[i]n dealing with an organization’s directors, officers, employees,

members, shareholders or other constituents, a lawyer shall explain the identity of the client

when the lawyer knows or reasonably should know that the organization’s interests are adverse

to those of the constituents with whom the lawyer is dealing.” Pa. R.P.C. No. 1.13(d).

Accordingly, when discussing the Supplier deal with any of the board members Able should

make it clear that his client is KI and that he represents KI and not any of the members

individually.

Able must resolve what action to take when he learns of conduct that is in violation of a

director’s legal obligation to the corporation when that action “is likely to result in substantial

injury to the organization.” RPC 1.13(b). RPC 1.13(b) provides that when a lawyer learns of

such conduct the lawyer should “proceed as is reasonably necessary in the best interest of the

organization.” Pa.R.P.C. No. 1.13(b). This subsection further provides that counsel should ask

the wrongdoers to reconsider, or, depending on the circumstances, suggest that they obtain

another opinion to confirm counsel’s position. If those actions are ineffective, the attorney

should take the matter to higher authorities in the organization, if possible. Pa.R.P.C. No.

1.13(b) & cmt. 3. If none of the foregoing are effective to resolve the issue, counsel may resign

in compliance with RPC 1.16. Pa.R.P.C. No. 1.13(c). When Able learned of the wrongdoing

that was and would continue to harm the corporation, he was obligated to try to resolve the issue

internally in order to protect the interests of his client, KI. Able took a first step when he

confronted Al. Al did nothing in response to Able’s advice other than tell Able to keep quiet

about the deal and threaten to fire Able and sue him if he did not keep his mouth shut.

Given Al’s reaction to Able’s advice, Able should determine whether he should discuss

the matter with the board as a whole. Rule 1.13 indicates that Able should take this step as the

board would be an authority higher than Al. See Pa.R.P.C. No. 1.13 cmt. 3. There is no way to

45

Page 50: JULY 2016 PENNSYLVANIA BAR EXAMINATION · JULY 2016 PENNSYLVANIA BAR EXAMINATION Essay Questions and Examiners’ Analyses and Performance Test Pennsylvania Board of Law Examiners

know how the board will react to Able’s advice, however, there is a good chance, given the fact

that the board members are beneficiaries of the improper deal and that they approved the deal,

that they will resolve not to take any action to rescind the deal. If this occurs, Able can resign or

consider informing Carl about the secret dealt. Clearly, the board has breached their fiduciary

duties to KI. Able’s duty is not to KI’s board members individually but, instead, is to KI.

One factor to consider with respect to disclosure to Carl is confidentiality. RPC 1.6

addresses confidentiality of information. Generally, an attorney may not disclose confidential

information without the client’s consent unless certain exceptions apply. “Except to the extent

that the client’s instructions or special circumstances limit that authority, a lawyer is impliedly

authorized to make disclosures about a client when appropriate in carrying out the

representation.” Pa.R.P.C. No. 1.6, cmt. 6. In this case, the information that Able has obtained,

is protected for the benefit of KI and not KI’s individual board members. Therefore, Able should

analyze the possible disclosure of the board’s action to Carl in the context of his duties to KI.

Able must ask if it would be in KI’s best interest for him to disclose the deal to Carl given that

Carl is a disinterested shareholder whose interest would align with that of KI. Able is faced with

a situation where he has no one to report the misconduct of the board to other than to Carl. Thus,

if Able wants a person with an interest in the corporation who is not a perpetrator of the

misconduct to independently review the matter and determine how to proceed he would disclose

the board’s action to Carl.

Able must analyze what action would be in the best interest of the client. Under rule 1.13

the corporation is Able’s client and Able must act in a manner that serves its best interests. See

Pa.R.P.C. 1.13(a), (b). Given this objective, Able should proceed in a way to minimize the risk

of harm to KI. As Able has a duty to protect his client, here KI, and would impliedly be

authorized under RPC number 1.6(a) to disclose the deal to Carl in order to fulfill his duty of

representation to KI, he should disclose to Carl. Able, however, should limit his disclosure to

Carl and not extend it to the public generally as a public disclosure could have a negative impact

on KI with its suppliers and customers. See Pa.R.P.C. 1.13(b).

2. Carl should demand, in writing, that the KI board of directors authorize a suit

against the directors to recover all of the past and future improper “finder’s fees”

paid to the directors. If the KI’s board refuses to do so, Carl should file a

shareholder’s derivative action on behalf of KI to recover the finder’s fees.

A shareholder’s derivative action is “an action to enforce a secondary right brought by

one or more stockholders or members of a corporation or similar entity because the corporation

or entity refuses or fails to enforce rights which could be asserted by it.” Pa.R.C.P. No. 1506(a).

The right to bring the suit is a “secondary” right because it is an equitable action brought by a

shareholder to enforce a right of the corporation itself and not a personal, or primary, right of the

shareholder. Derivative suits developed as a way for shareholders to protect the interests of the

corporation from the malfeasance or misfeasance of its directors. A shareholder’s derivative

action is filed on behalf of the corporation and not to enforce a direct claim held by the

shareholder. See, 3 W. Edward Sell & William H. Clark, Jr., Pennsylvania Business

Corporations, §1782.2 (Rev. 2d Ed. 1998).

46

Page 51: JULY 2016 PENNSYLVANIA BAR EXAMINATION · JULY 2016 PENNSYLVANIA BAR EXAMINATION Essay Questions and Examiners’ Analyses and Performance Test Pennsylvania Board of Law Examiners

A shareholder’s derivative suit may be filed if the party filing the action meets certain

criteria under section 1782 of the Pennsylvania Business Corporation Law of 1988 (hereinafter

the “BCL”). 15 Pa.C.S.A. § 1782. First, the plaintiff in the action must have been a shareholder

at the time of the transaction giving rise to the suit and also must be a current shareholder. 15

Pa.C.S.A. § 1782(a); Pa.R.C.P. No. 1506(a)(1). Second, the plaintiff, generally, must have made

a demand upon the board of directors to address the problem. Cuker v. Mikalauskas, 547 Pa.

600, 613, 692 A.2d 1042, 1049, app. § 7.03 (1997); Pa.R.C.P. No. 1506. Third, the board of

directors of the corporation and the corporation must have refused the demand and failed to take

corrective action. Id.; See, Pennsylvania Business Corporations, § 1782.3.

In this case, the KI board has clearly breached its fiduciary duty to the corporation by

entering into the secret deal with a supplier that resulted in them receiving finder’s fees from the

increased profits derived by the supplier as a result of the excess charges to KI. The higher rates

being paid by KI to this supplier have materially impacted KI’s profits and will diminish

distributions to shareholders, including Carl. Although, Carl does not having a direct claim

against the board members he should take the necessary steps to assert the right of KI to recover

the improper finder’s fees from the board members. Carl is permitted to file a derivative action

on behalf of KI because he was a shareholder at the time of the improper transaction and

continues to be a shareholder at the time of the suit.

Carl, however, should send a written notice to KI’s board, demanding that the board

cancel the supplier deal and require all board members to return all finder’s fees to KI or sue the

board members to recover the improper finder’s fees. If the board refuses the demand, Carl

should file suit on behalf of KI. This demand would satisfy the requirement of Pennsylvania

Rule of Civil Procedure No. 1506(a)(2) that requires the filer of a shareholder derivative action

to state in the complaint “the efforts made to secure enforcement by the corporation or similar

entity or the reason for not making any such efforts.” The Pennsylvania Supreme Court has

recognized an “irreparable injury” exception to the demand requirement. See, Cuker, 547 Pa. at

616, 692 A.2d at 1049, app. §7.03. Under Cuker, if Carl could establish that making a written

demand in this case would cause the corporation irreparable injury, the demand would be

unnecessary. Although Carl might proceed without making the demand, and plan to argue if

challenged that the demand was not required, sending the written notice coupled with the failure

of the board to act, eliminates this potential procedural defense and is not unduly burdensome on

Carl.

If after Carl’s demand the board disagrees with Carl and fails to act, Carl should direct

his counsel to prepare and file a shareholder’s derivative action on behalf of KI naming the

board, and KI as defendants. See, Pa.R.C.P. No. 2177. The complaint should reference the

demand made by Carl, should indicate that Carl is presently a shareholder and was a shareholder

of KI at the time the deal was made and the finder’s fees were paid, and claim that the board

members have breached their fiduciary duties to KI by making the deal and accepting the

benefits thereof.

47

Page 52: JULY 2016 PENNSYLVANIA BAR EXAMINATION · JULY 2016 PENNSYLVANIA BAR EXAMINATION Essay Questions and Examiners’ Analyses and Performance Test Pennsylvania Board of Law Examiners

3. Boxes will be able to enforce the interest provision in its confirming memorandum if

KI fails to make payment within thirty (30) days of delivery if such a provision is

reasonable within the industry.

The Pennsylvania Uniform Commercial Code, Article 2, Sales (the “Code”) defines

merchants in relevant part as “[a] person who: (1) deals in goods of the kind; or (2) otherwise by

his occupation holds himself out as having knowledge or skill peculiar to the practices or goods

involved in the transaction.” 13 Pa.C.S.A. § 2104. KI and Boxes are both merchants under the

Code as Boxes sells packaging materials and boxes and KI routinely uses those materials and

boxes to package and sell the goods that it produces. The confirming memo faxed by Boxes

accepted KI’s verbal order to purchase boxes even though it added additional terms. See 13

Pa.C.S.A. § 2207(a). The issue then becomes one of the applicability of the interest provision,

not discussed by the merchants, that was included in the confirming memo sent by Boxes.

Section 2207 of the Code addresses this situation. It provides, inter alia, as follows:

Additional terms in acceptance or confirmation

(a) General rule.--A definite and seasonable expression of acceptance or a

written confirmation which is sent within a reasonable time operates as an

acceptance even though it states terms additional to or different from those

offered or agreed upon, unless acceptance is expressly made conditional on assent

to the additional or different terms.

(b) Effect on contract.--The additional terms are to be construed as proposals for

addition to the contract. Between merchants such terms become part of the

contract unless:

(1) the offer expressly limits acceptance to the terms of the offer;

(2) they materially alter it; or

(3) notification of objection to them has already been given or is given

within a reasonable time after notice of them is received.

13 Pa.C.S.A. § 2207(a)-(b). Under section 2207, a written confirmation accepting an offer

operates as an acceptance although it contains additional terms. If the confirmation is directed to

a non-merchant the new terms are proposals for addition to the contract. If between merchants,

however, the terms are proposals that become part of the contract unless the offer limited

acceptance to the terms of the offer, the terms materially alter the contract, or there is an

objection to the terms within a reasonable time.

Subpart (b)(2) of section 2207 is particularly relevant to the facts in this case. The

language adding interest if payment is not made within thirty (30) days is an additional term. It

was not discussed when Al and the representative of Boxes spoke by telephone. The issue then

becomes, given that the transaction involves two merchants, whether this additional term

48

Page 53: JULY 2016 PENNSYLVANIA BAR EXAMINATION · JULY 2016 PENNSYLVANIA BAR EXAMINATION Essay Questions and Examiners’ Analyses and Performance Test Pennsylvania Board of Law Examiners

49

materially alters the contract. The comments to section 2207 suggest that an additional term will

materially alter the contract if it will result in surprise or hardship if incorporated into the

contract. 13 Pa.C.S.A. § 2207 cmts. 3-4. The comments to 2207 also provide that a clause

providing for interest on an overdue invoice is a clause that does not materially alter a contract if

it is within the range of trade practice. 13 Pa.C.S.A. §2207 cmt. 5. A thirty (30) day time for

interest to attach and the interest rate of 1.5% monthly, however, appears to be reasonable and

would most likely be consistent with trade practice for similarly situated parties. See, James J.

White & Robert S. Summers, Uniform Commercial Code, § 1-3 at 15 (4th ed. 1995); Herzog Oil

Field Serv, Inc. v. Otto Torpedo Co., 570 A.2d 549, 551 (Pa. Super. 1990) (finding a thirty day

time for interest to attach and an interest rate of 1.5% monthly to be reasonable).

4. Betty’s sister could claim that KI breached the express warranty that its salesman

made to Betty and should be able to recover as a third party beneficiary of the

warranty who suffered damages as a result of the breach.

Section 2313 of the Code provides that a seller of goods creates an express warranty by

“[a]ny affirmation of fact or promise made by the seller to the buyer which relates to the goods

and becomes part of the basis of the bargain.” 13 Pa.C.S.A. § 2313(a)(1). An affirmation of fact

is generally a statement that is specific and verifiable and usually is in the nature of a description

of the goods being sold. “Any affirmation of fact or any promise by the seller relating to the

goods is an express warranty if the natural tendency of such affirmation or promise is to induce

the buyer to purchase the goods . . . .” James J. White & Robert S. Summers, Uniform

Commercial Code, § 9-5 at 338 (4th ed. 1995). The affirmation can be found to be part of the

basis of the bargain if the buyer relies on the statement when deciding whether or not to purchase

the good, although actual reliance need not be show if the affirmation was used to describe the

good and induced the buyer to buy. Id. An express warranty warrants “that the goods shall

conform to the affirmation or promise.” 13 Pa. C.S.A. §2313(a)(1).

The facts provide that Betty went to KI’s showroom seeking a pottery bowl that she

could use in her microwave oven. She specifically asked KI’s salesman if KI had bowls that

could be used to heat food in a microwave. The salesman clearly and explicitly pointed out

bowls to Betty and guaranteed that any of the bowls that he suggested would be suitable and safe

for use in a microwave oven and, given Betty’s inquiry and her apparent reliance on what was

said, this statement was clearly part of the basis of the bargain. When Betty’s sister placed the

bowl in the microwave and began heating the cheese dip the bowl burst, shattering the glass door

of the microwave and spewing hot cheese and glass into the sister’s face. It is obvious that the

bowl did not perform as KI’s salesman had promised. Clearly, Betty would have a basis to assert

a breach of express warranty claim against KI because the goods did not conform to the

affirmation made by the salesman.

Although it is clear that Betty could assert a breach of express warranty claim, the issue

becomes whether Betty’s sister, who did not buy the bowl and who is not in contractual privity

with KI, could assert a breach of express warranty claim. Section 2318 of the Code addresses

warranty claims by third party beneficiaries. Section 2318 states, “[t]he warranty of a seller

whether express or implied extends to any natural person who is in the family or household of

his buyer or who is a guest in his home if it is reasonable to expect that such person may use,

Page 54: JULY 2016 PENNSYLVANIA BAR EXAMINATION · JULY 2016 PENNSYLVANIA BAR EXAMINATION Essay Questions and Examiners’ Analyses and Performance Test Pennsylvania Board of Law Examiners

consume or be affected by the goods and who is injured in person by breach of the warranty.” 13

Pa.C.S.A. § 2318. The express warranty claim that arises under the facts could be asserted by

Betty’s sister as (1) she was a guest in Betty’s home, (2) it is reasonable to expect that she might

use or be affected by the use of the bowl in a microwave, and (3) she was injured (suffered

personal injury) by the failure of bowl. Accordingly, Betty’s sister could assert a breach of

express warranty claim directly against KI.

50

Page 55: JULY 2016 PENNSYLVANIA BAR EXAMINATION · JULY 2016 PENNSYLVANIA BAR EXAMINATION Essay Questions and Examiners’ Analyses and Performance Test Pennsylvania Board of Law Examiners

Question No. 6: Grading Guidelines

1. Professional Responsibility—Duties of Corporate Counsel upon Discovery of

Wrongdoing

Comments: The candidates should discuss who the client is and who counsel owes his duties to.

The candidates also should discuss the action to be taken once the wrongdoing is discovered and

whether or not counsel may disclose the wrongdoing to shareholders.

5 points

2. Corporations—Shareholder Derivative Action

Comments: The candidates should recognize the availability of a derivative action and the

procedural requirements that must be met before the action may be filed.

5 points

3. Sales—Confirming Memo Adding New Terms Between Merchants

Comments: The candidates should recognize that the stated transaction involves two merchants

and that a confirming memo accepting an offer acts as an acceptance even with a new term and

that the new term becomes part of the deal between merchants if it does not materially alter the

contract.

5 points

4. Sales—Extension of Express Warranty to Third Party Beneficiary

Comments: The candidates should discuss the creation of the express warranty and the

availability of the warranty to the third party under these facts.

5 points

51

Page 56: JULY 2016 PENNSYLVANIA BAR EXAMINATION · JULY 2016 PENNSYLVANIA BAR EXAMINATION Essay Questions and Examiners’ Analyses and Performance Test Pennsylvania Board of Law Examiners

PT

Supreme Court of Pennsylvania

Pennsylvania Board of Law Examiners

Pennsylvania Bar Examination

Fe

PERFORMANCE TEST

February 24, 2015

Use GRAY covered book for your answer to the Performance Test.

© 2015 Pennsylvania Board of Law Examiners

PLACE BAR CODED APPLICANT LABEL HERE

Question Number 3

on SofTest

52

Page 57: JULY 2016 PENNSYLVANIA BAR EXAMINATION · JULY 2016 PENNSYLVANIA BAR EXAMINATION Essay Questions and Examiners’ Analyses and Performance Test Pennsylvania Board of Law Examiners

TABLE OF CONTENTS

FILE

Memorandum to applicant outlining task ........................................................................................1 Memorandum on preparing motions ................................................................................................2 Complaint .........................................................................................................................................3 Notice of removal ............................................................................................................................6 LIBRARY

Statutes

Pa.R.C.P. No. 1007 ..........................................................................................................................8 Pa.R.C.P. No. 1017 ..........................................................................................................................8 28 U.S.C. § 1332 ..............................................................................................................................8 28 U.S.C. § 1441 ..............................................................................................................................8 28 U.S.C. § 1446 ..............................................................................................................................9 28 U.S.C. § 1447 ..............................................................................................................................9 Case law

Lincoln Property Co. V. Roche ....................................................................................................... 10

Warner v. Mut. Life Ins. Co. ........................................................................................................... 14

Atcovitz v. Gulph Mills Tennis Club ............................................................................................... 17

53

Page 58: JULY 2016 PENNSYLVANIA BAR EXAMINATION · JULY 2016 PENNSYLVANIA BAR EXAMINATION Essay Questions and Examiners’ Analyses and Performance Test Pennsylvania Board of Law Examiners

FILE

54

Page 59: JULY 2016 PENNSYLVANIA BAR EXAMINATION · JULY 2016 PENNSYLVANIA BAR EXAMINATION Essay Questions and Examiners’ Analyses and Performance Test Pennsylvania Board of Law Examiners

MEMORANDUM

TO: Applicant FROM: Luke Flickinger CLIENT: Peter Pratt, Client # 060395 SUBJECT: Assignment to Draft Motion to Remand We represent Peter Pratt concerning his claims against his insurance company, First Insurance Company, and its adjuster, Deborah Dunn, individually, for state-law-based insurance bad faith and ordinary negligence. On June 1, 2015, we initiated an action against both defendants by properly filing a writ of summons with the prothonotary of the Court of Common Pleas of North County, Pennsylvania. We served the writ of summons on the same day. We properly filed the complaint in the action on June 1, 2016, with the prothonotary of the Court of Common Pleas of North County, Pennsylvania. Both defendants were properly served with the complaint on June 6, 2016. Defendants filed a notice of removal on July 10, 2016, with the United States District Court for the Northern District of Pennsylvania, removing the action from state court to federal court. The Northern District of Pennsylvania encompasses, among other counties, North County, Pennsylvania. Your assignment is to draft a motion to remand our case to the Court of Common Pleas of North County, Pennsylvania. I have attached our complaint against the defendants (without the requisite notice to defend or certificates of service, although those documents were properly filed and served) and defendants’ notice of removal, along with excerpts from relevant rules, cases, and statutes. Please include any and all reasons for remand, including reasons in the alternative. Do not rely on your personal knowledge of these issues or procedures, or on cases, statutes, or rules that are not included in the attached library. Instead, base your facts and legal authority in the motion only upon the documents provided in the file and library. The Northern District of Pennsylvania has recently adopted a new rule requiring motions to include any necessary facts and legal authority in lieu of filing both a motion and a brief in support of a motion. You should draft the motion using the format set forth in the attached memorandum setting forth guidelines for drafting a motion. Do not include a caption; do not sign the document; and do not include your name or address, as I will have that information added before filing. I have spoken with opposing counsel and I will file the proper certificate of non-concurrence along with the motion. I will also draft the proposed order. Formal Bluebook citations are not required as I will have my paralegal check all citations before filing; however, you should sufficiently identify any document, rule, statute, or case such that I will know to what authority you are referring in your motion.

55

Page 60: JULY 2016 PENNSYLVANIA BAR EXAMINATION · JULY 2016 PENNSYLVANIA BAR EXAMINATION Essay Questions and Examiners’ Analyses and Performance Test Pennsylvania Board of Law Examiners

MEMORANDUM

TO: Applicant FROM: Luke Flickinger DATE: July 26, 2016 SUBJECT: Preparing Motions Applicants should use the following guidelines when drafting a motion for my review.

1. Place the title of the motion at the top of the motion and include a reference to the party bringing the motion.

2. Include an introductory paragraph advising the court of the purpose for filing the motion and the authority under which the motion is brought.

3. Each legal argument should be addressed under separate headings, just as each cause of action is brought in a separate section of the complaint below.

4. A motion should set forth all relevant facts and legal authority necessary to support the

arguments in support of the relief requested.

5. Set forth the legal authority (statutes, rules, and caselaw) upon which you are relying to support your argument, including each distinct legal principle, in separately numbered paragraphs within the body of a motion. Each legal principle should be supported by a citation to the legal authority setting forth the principle. Formal Bluebook citations are not required.

6. Set forth in separately numbered paragraphs each fact that supports your arguments including citation to the documents in the file supporting that fact. Formal Bluebook citations are not required.

7. Necessary factual averments may be incorporated into other sections of the motion by reference.

8. Include as many paragraphs as you believe are necessary to set forth the facts and law that you deem appropriate to support your arguments.

9. A motion should be concluded with a request for the relief being sought. Do not number the concluding paragraph and start it with “WHEREFORE.”

56

Page 61: JULY 2016 PENNSYLVANIA BAR EXAMINATION · JULY 2016 PENNSYLVANIA BAR EXAMINATION Essay Questions and Examiners’ Analyses and Performance Test Pennsylvania Board of Law Examiners

Peter Pratt, : In the Court of Common Pleas of North County, 1 Main Street : Pennsylvania Notown, PA 99999 :

Plaintiff : Civil Action – Law : Jury Trial Demanded v. : : No. 2016 CIV 11052010 First Insurance Company : 2775 Money Road : Northbrook, IL 60062 : : and : : Deborah Dunn, an individual : 2 Center Street : Realtown, PA 99998 : Defendants :

COMPLAINT

Now comes the Plaintiff, Peter Pratt, by and through his attorneys, Smith & Jones, P.C., and for

his Complaint against First Insurance Company and Deborah Dunn alleges:

1. The Plaintiff, Peter Pratt, is a competent adult individual and a citizen of Pennsylvania,

who currently, and at all times relevant hereto, resides at 1 Main Street, Notown, North County,

Pennsylvania, 99999.

2. The Defendant, First Insurance Company (hereinafter “First”), is a company incorporated

in Illinois and has its principal offices situated at 2775 Money Road, Northbrook, Illinois, 60062. At all

times relevant hereto, First regularly conducted business in North County, Pennsylvania.

3. The Defendant, Deborah Dunn (hereinafter “Dunn”), is a competent adult individual and

a citizen of Pennsylvania, who is, and throughout all relevant time periods, has been an employee of

First, and who currently, and at all times relevant hereto, resides at 2 Center Street, Realtown, North

County, Pennsylvania, 99998.

4. Plaintiff was the driver of a vehicle that was lawfully stopped at a red traffic light on

January 2, 2015, when said vehicle was struck from behind by another vehicle.

57

Page 62: JULY 2016 PENNSYLVANIA BAR EXAMINATION · JULY 2016 PENNSYLVANIA BAR EXAMINATION Essay Questions and Examiners’ Analyses and Performance Test Pennsylvania Board of Law Examiners

5. Plaintiff was an insured of First on the date of the accident, when Plaintiff sustained

severe, life-threatening injuries, of which Defendants were notified and kept up to date on throughout

Plaintiff’s hospitalization and treatment (which is ongoing), and which has left Plaintiff paralyzed from

the neck down.

COUNT I – Pratt v. First – Negligence and Bad Faith

6. Plaintiff incorporates each and every allegation of this Complaint by reference as if fully

set forth herein at length.

7. First had duties to properly handle Plaintiff’s claim for insurance benefits under common

law and under Pennsylvania’s insurance bad faith statute.

8. First assigned the handling of Plaintiff’s claim to Dunn.

9. First hired Dunn despite knowing that she had been fired from seven other insurance

companies for mishandling claims.

10. Although Plaintiff’s injuries warranted immediate payment of the One Hundred

Thousand Dollars ($100,000) underinsured motorist policy limits, First improperly refused to pay the

claim.

11. First negligently failed to pay Plaintiff’s claim to which he was entitled; said failure being

the direct and proximate cause of significant and actual financial harm to Plaintiff.

12. First’s mishandling of Plaintiff’s claim violated Pennsylvania’s insurance bad faith

statute.

13. Plaintiff is entitled to payment of the insurance policy limits, plus punitive damages, as a

result of First’s negligence and bad faith.

COUNT II – Pratt v. Dunn - Negligence

14. Plaintiff incorporates each and every allegation of this Complaint by reference as if fully

set forth herein at length.

58

Page 63: JULY 2016 PENNSYLVANIA BAR EXAMINATION · JULY 2016 PENNSYLVANIA BAR EXAMINATION Essay Questions and Examiners’ Analyses and Performance Test Pennsylvania Board of Law Examiners

15. As the adjuster assigned to Plaintiff’s insurance claim, Dunn had a duty to properly

handle Plaintiff’s claim, pursuant to Pennsylvania law.

16. Dunn’s individual duty to Plaintiff was independent of the duties First had to Plaintiff.

17. Dunn failed to perform her duty to properly handle Plaintiff’s claim, and instead

frequently and incorrectly cited to someone else’s medical records in letters sent to Plaintiff denying his

claim, among other actions demonstrating a complete lack of regard for Plaintiff and his injuries.

18. Dunn never talked to Plaintiff, and refused to do so; she negligently mishandled

Plaintiff’s claim at every step.

19. Dunn negligently failed to adjust Plaintiff’s claim and pay the policy limits to which he

was entitled; said failure causing significant financial harm to Plaintiff.

20. Dunn’s failure to properly adjust Plaintiff’s claim and pay the policy limits is the direct

cause and proximate cause of Plaintiff’s harm; had she properly adjusted the claim, it would have been

paid upon Plaintiff’s demand.

21. Dunn’s mishandling of Plaintiff’s claim was negligent.

22. Plaintiff is entitled to payment of the insurance policy limits, plus punitive damages, as a

result of Dunn’s negligence.

WHEREFORE, Plaintiff requests this Honorable Court enter judgment in Plaintiff’s favor and

against Defendants in an amount of at least $100,000 along with punitive damages, costs, and any other

amounts deemed reasonable by the Court.

Respectfully Submitted,

Dated: June 1, 2016 Luke Flickinger

59

Page 64: JULY 2016 PENNSYLVANIA BAR EXAMINATION · JULY 2016 PENNSYLVANIA BAR EXAMINATION Essay Questions and Examiners’ Analyses and Performance Test Pennsylvania Board of Law Examiners

Peter Pratt, : In the United States District Court for the Plaintiff : Northern District of Pennsylvania v. : : First Insurance Company : : and : : Deborah Dunn, an individual : Defendants : TO: CLERK OF COURT IN THE UNITED STATES DISTRICT COURT FOR THE

NORTHERN DISTRICT OF PENNSYLVANIA

NOTICE OF REMOVAL

Defendants, First Insurance Company and Deborah Dunn, hereby file Notice of Removal of this

case from the Court of Common Pleas of North County, Pennsylvania, where it is now pending, to the United States District Court for the Northern District of Pennsylvania on the basis of diversity jurisdiction, and in support thereof aver as follows:

1. Plaintiff filed a Complaint in this matter on June 1, 2016, in the Court of Common Pleas of North

County, Pennsylvania, docketed at 2016 CIV 11052010.

2. The Complaint was served on both Defendants on June 6, 2016.

3. The Complaint states that Defendant, First Insurance Company, is a company incorporated in

Illinois, with its principal offices situated at 2775 Money Road, Northbrook, Illinois, 60062.

4. The Complaint states that Plaintiff, Peter Pratt, is a competent adult individual and a citizen of

Pennsylvania, who currently resides at 1 Main Street, Notown, North County, Pennsylvania,

99999.

5. The Complaint states that the amount in controversy is in excess of Seventy-Five Thousand

Dollars ($75,000.00), exclusive of costs and interest, as Plaintiff has demanded the insurance

policy limits of One Hundred Thousand Dollars ($100,000.00), plus punitive damages, costs, and

other amounts in connection with his claims.

60

Page 65: JULY 2016 PENNSYLVANIA BAR EXAMINATION · JULY 2016 PENNSYLVANIA BAR EXAMINATION Essay Questions and Examiners’ Analyses and Performance Test Pennsylvania Board of Law Examiners

6. As such, the above described civil action is one in which this Honorable Court has jurisdiction

pursuant to the provisions of 28 U.S.C. § 1332 based upon the fact that there exists diversity of

citizenship between the Plaintiff, Peter Pratt, and First Insurance Company, and the amount in

controversy exceeds $75,000.00.

7. Dunn is not properly a party to this action as she was only joined as a party to defeat diversity.

8. Plaintiff’s claims against Dunn are not colorable under Pennsylvania law, and this Honorable

Court should therefore disregard her citizenship for the purpose of determining diversity of

citizenship.

10. The present lawsuit is accordingly removable from the state court to this Honorable Court

pursuant to 28 U.S.C. § 1441 and § 1446.

11. Copies of all process, pleadings and other orders which have been received by Defendants in this

action are filed herewith.

WHEREFORE, Defendants respectfully request that they may affect the removal of this action

from the Court of Common Pleas of North County, Pennsylvania, to the United States District Court for

the Northern District of Pennsylvania.

Respectfully Submitted,

William C. Forrester Attorney for Defendants Dated: July 10, 2016

61

Page 66: JULY 2016 PENNSYLVANIA BAR EXAMINATION · JULY 2016 PENNSYLVANIA BAR EXAMINATION Essay Questions and Examiners’ Analyses and Performance Test Pennsylvania Board of Law Examiners

LIBRARY

62

Page 67: JULY 2016 PENNSYLVANIA BAR EXAMINATION · JULY 2016 PENNSYLVANIA BAR EXAMINATION Essay Questions and Examiners’ Analyses and Performance Test Pennsylvania Board of Law Examiners

Pa.R.C.P. No. 1007. Commencement of Action An action may be commenced by filing with the prothonotary (1) a praecipe for a writ of summons, or (2) a complaint. Pa.R.C.P. No. 1017. Pleadings Allowed (a) . . . the pleadings in an action are limited to (1) a complaint and an answer thereto,

* * * (2) a reply if the answer contains new matter, a counterclaim or a cross-claim, (3) a counter-reply if the reply to a counterclaim or cross-claim contains new matter, (4) a preliminary objection and a response thereto. 28 U.S.C. § 1332 – Diversity of citizenship; amount in controversy; costs

(a) The district courts shall have original jurisdiction of all civil actions where the matter in controversy exceeds the sum or value of $75,000, exclusive of interest and costs, and is between—

(1) citizens of different States . . . . * * *

(c) For the purposes of this section and section 1441 of this title – (1) a corporation shall be deemed to be a citizen of every State and foreign state by which it has

been incorporated and of the State or foreign state where it has its principal place of business . . . . 28 U.S.C. § 1441 – Removal of civil actions (a) Generally.— Except as otherwise expressly provided by Act of Congress, any civil action brought in a State court of which the district courts of the United States have original jurisdiction, may be removed by the defendant or the defendants, to the district court of the United States for the district and division embracing the place where such action is pending. (b) Removal Based on Diversity of Citizenship.—

* * * (2) A civil action otherwise removable solely on the basis of the jurisdiction under section 1332(a) of this title may not be removed if any of the parties in interest properly joined and served as defendants is a citizen of the State in which such action is brought.

63

Page 68: JULY 2016 PENNSYLVANIA BAR EXAMINATION · JULY 2016 PENNSYLVANIA BAR EXAMINATION Essay Questions and Examiners’ Analyses and Performance Test Pennsylvania Board of Law Examiners

28 U.S.C. § 1446 – Procedure for removal of civil actions

(a) Generally.— A defendant or defendants desiring to remove any civil action from a State court shall file in the district court of the United States for the district and division within which such action is pending a notice of removal . . . containing a short and plain statement of the grounds for removal . . . . (b) Requirements; generally.— (1) The notice of removal of a civil action or proceeding shall be filed within 30 days after the receipt by the defendant, through service or otherwise, of a copy of the initial pleading setting forth the claim for relief upon which such action or proceeding is based . . . .

28 U.S.C. §1447 – Procedure after removal generally

* * * (c) A motion to remand the case on the basis of any defect other than lack of subject matter jurisdiction must be made within 30 days after the filing of the notice of removal under section 1446(a).

64

Page 69: JULY 2016 PENNSYLVANIA BAR EXAMINATION · JULY 2016 PENNSYLVANIA BAR EXAMINATION Essay Questions and Examiners’ Analyses and Performance Test Pennsylvania Board of Law Examiners

546 U.S. 81 (2005)

LINCOLN PROPERTY CO., et al., Petitioners,

v.

Christophe ROCHE et ux.

***

This case concerns 28 U.S.C. § 1441, which authorizes the removal of civil actions from state

court to federal court when the action initiated in state court is one that could have been brought,

originally, in a federal district court. § 1441(a). When federal-court jurisdiction is predicated on the

parties' diversity of citizenship, see § 1332, removal is permissible "only if none of the parties in interest

properly joined and served as defendants is a citizen of the State in which [the] action [was] brought." §

1441(b).

Christophe and Juanita Roche, plaintiffs below, respondents here, are citizens of Virginia. They

commenced suit in state court against diverse defendants, including Lincoln Property Company

(Lincoln), a corporation chartered and having its principal place of business in Texas. The defendants

removed the litigation to a Federal District Court where, after discovery proceedings, they successfully

moved for summary judgment. Holding the removal improper, the Court of Appeals instructed remand

of the action to state court. (citation omitted) The appellate court so ruled on the ground that the Texas

defendant failed to show the nonexistence of an affiliated Virginia entity that was the "real party in

interest." (citation omitted)

We reverse the judgment of the Court of Appeals. Defendants may remove an action on the basis

of diversity of citizenship if there is complete diversity between all named plaintiffs and all named

defendants, and no defendant is a citizen of the forum State. It is not incumbent on the named

defendants to negate the existence of a potential defendant whose presence in the action would destroy

diversity. (footnote omitted)

* * *

Christophe and Juanita Roche leased an apartment in the Westfield Village complex in Fairfax

County, Virginia. About a year after moving in, they discovered evidence of toxic mold, which the

inspection report linked to hair loss, headaches, irritation of the respiratory tract, fatigue, and dermatitis.

(citation omitted) . . . The Roches moved out of their apartment for the remediation process, leaving

65

Page 70: JULY 2016 PENNSYLVANIA BAR EXAMINATION · JULY 2016 PENNSYLVANIA BAR EXAMINATION Essay Questions and Examiners’ Analyses and Performance Test Pennsylvania Board of Law Examiners

their personal belongings in the care of Lincoln, the designated property manager of Westfield Village,

and the mold treatment firm. (citation omitted)

Some months later, the Roches commenced suit, filing two substantially similar complaints in

the Circuit Court for Fairfax County, Virginia. (citation omitted) Both complaints asserted serious

medical ailments from the Roches' year-long exposure to toxic mold, and sought damages under

multiple headings, including negligence, breach of contract, actual fraud, constructive fraud, and

violations of Virginia housing regulations. (citation omitted) In addition, the Roches alleged loss, theft,

or destruction of their personal property. (citation omitted) Regarding these losses, they sought

damages for conversion and infliction of emotional distress. (citation omitted)

* * *

Defendants timely removed the twin cases to the United States District Court for the Eastern

District of Virginia, invoking that court's diversity-of-citizenship jurisdiction. . . . The notice of removal

described Lincoln as a Texas corporation with its principal place of business in Texas, INVESCO as a

Delaware corporation with its principal place of business in Georgia, and State of Wisconsin Investment

Board as an independent agency of Wisconsin. . . . In their consolidated federal-court complaint, the

Roches identified themselves as citizens of Virginia and Lincoln as a corporation headquartered in

Texas, just as they did in their state-court complaints. (citation omitted) (footnote omitted) Further,

they stated affirmatively that the federal court "has jurisdiction of this matter."

* * *

After discovery, the parties cross-moved for summary judgment.

* * *

Six days after the District Court granted defendants' motion for summary judgment, but before

final judgment was entered, the Roches moved to remand the case to the state court, alleging for the first

time the absence of federal subject-matter jurisdiction. (footnote omitted) Specifically, the Roches

alleged that Lincoln "is not a Texas Corporation, but a Partnership with one of its partners residing in the

Commonwealth of Virginia." (citation omitted) (footnote omitted) The District Court denied the

remand motion, concluding that Lincoln is a Texas corporation and that removal was proper because the

requisite complete diversity existed between all plaintiffs and all defendants. (citation omitted)

66

Page 71: JULY 2016 PENNSYLVANIA BAR EXAMINATION · JULY 2016 PENNSYLVANIA BAR EXAMINATION Essay Questions and Examiners’ Analyses and Performance Test Pennsylvania Board of Law Examiners

The Court of Appeals for the Fourth Circuit reversed and instructed the District Court to remand

the case to the state court. (citation omitted) Although recognizing that Lincoln is a Texas citizen and a

proper party to the action . . . the Court of Appeals observed that "Lincoln operates under many different

structures[.]" (citation omitted) Describing Lincoln as "the nominal party and ultimate parent

company," the appellate court suspected that an unidentified "Virginia subsidiary, be it a partnership,

corporation or otherwise, rather than the Texas parent" was "the real and substantial party in interest."

(citation omitted) Lincoln, the party invoking federal-court jurisdiction, had not demonstrated the

nonexistence of "the Virginia sub-'partnership,'" the Court of Appeals reasoned (citation omitted) and

therefore had not met its burden of establishing diversity[.] (citation omitted)

* * *

The Court of Appeals correctly identified Lincoln as a proper party to the action, but it erred in

insisting that some other entity affiliated with Lincoln should have been joined as a codefendant, and

that it was Lincoln's obligation to name that entity and show that its joinder would not destroy diversity.

We stress, first, that, at this stage of the case, the existence of complete diversity between the

Roches and Lincoln is not in doubt. The Roches, both citizens of Virginia, acknowledge that Lincoln is

indeed a corporation, not a partnership, and that Lincoln is chartered in and has its principal place of

business in Texas. (citation omitted) . . . Accordingly, for jurisdictional purposes, Lincoln is a citizen of

Texas and of no other State. 28 U.S.C. § 1332(c)(1) ("a corporation shall be deemed to be a citizen of

any State by which it has been incorporated and of the State where it has its principal place of

business").

We turn now to the reasons why the Fourth Circuit erred in determining that diversity

jurisdiction was not proved by the removing parties. (citation omitted) The principal federal statute

governing diversity jurisdiction, 28 U.S.C. § 1332, gives federal district courts original jurisdiction of all

civil actions "between . . . citizens of different States" where the amount in controversy exceeds

$75,000. § 1332(a)(1). (footnote omitted) Since Strawbridge v. Curtiss . . . we have read the statutory

formulation "between . . . citizens of different States" to require complete diversity between all plaintiffs

and all defendants. . . .

While § 1332 allows plaintiffs to invoke the federal courts' diversity jurisdiction, § 1441 gives

defendants a corresponding opportunity. Section 1441(a) states: "Except as otherwise expressly

provided by Act of Congress, any civil action brought in a State court of which the district courts of the

67

Page 72: JULY 2016 PENNSYLVANIA BAR EXAMINATION · JULY 2016 PENNSYLVANIA BAR EXAMINATION Essay Questions and Examiners’ Analyses and Performance Test Pennsylvania Board of Law Examiners

United States have original jurisdiction, may be removed by the defendant or the defendants, to the

district court of the United States for the district and division embracing the place where such action is

pending." The scales are not evenly balanced, however. An in-state plaintiff may invoke diversity

jurisdiction, but § 1441(b) bars removal on the basis of diversity if any "part[y] in interest properly

joined and served as [a] defendan[t] is a citizen of the State in which [the] action is brought." (footnote

omitted) In the instant case, Virginia plaintiffs Christophe and Juanita Roche joined and served no

Virginian as a party defendant. Hence the action qualified for the removal defendants effected.

* * *

For the reasons stated, the judgment of the United States Court of Appeals for the Fourth Circuit

is reversed, and the case is remanded for further proceedings consistent with this opinion.

It is so ordered.

68

Page 73: JULY 2016 PENNSYLVANIA BAR EXAMINATION · JULY 2016 PENNSYLVANIA BAR EXAMINATION Essay Questions and Examiners’ Analyses and Performance Test Pennsylvania Board of Law Examiners

998 F. Supp. 592 (1998)

Ellice WARNER

v.

The MUTUAL LIFE INSURANCE COMPANY OF NEW YORK, et al.

United States District Court, E.D. Pennsylvania.

***

Facts

Plaintiff brought this suit for breach of contract against all of the defendants and for bad faith

under 42 Pa.C.S. § 8371 against the Mutual Life Insurance Company of New York ("MONY").

According to the complaint, on May 8, 1991 defendant Milnazik, acting as the agent of defendants

Marciano and MONY, went to plaintiff's office and conducted a personal history interview, and

prepared and submitted plaintiff's application for a disability income insurance policy to be issued by

MONY. (citation omitted) MONY subsequently issued Plaintiff a disability income insurance policy on

May 20, 1991, which policy had an annual premium of $1,992.20. (citation omitted)

Five years later, in May, 1996, Ms. Warner submitted a claim to MONY for basic monthly

income benefits under the disability policy asserting that she was totally disabled from her regular

occupation due to chronic pain syndrome and fibromyalgia. (citation omitted) Following defendants'

denial of this claim in March, 1997, plaintiff brought this suit in the Court of Common Pleas of

Philadelphia County in August, 1997. On August 22, Defendant MONY removed the action to this

Court on the basis of diversity jurisdiction. On September 16, 1998, Plaintiff moved for remand on the

grounds that, contrary to defendant's notice of removal, the district court does not have diversity

jurisdiction given that M.J. Milnazik & Associates is a citizen of Pennsylvania.

Discussion

The principles and procedures governing removal of actions from a state court to a federal forum

are set forth in 28 U.S.C. § 1441 . . . .

***

Thus, as § 1441(a)'s language indicates, removal under that section is proper only if the federal

district court would have had original jurisdiction if the case was filed in federal court. (citation omitted)

This jurisdictional prerequisite to removal is an absolute, nonwaivable requirement in recognition of the

69

Page 74: JULY 2016 PENNSYLVANIA BAR EXAMINATION · JULY 2016 PENNSYLVANIA BAR EXAMINATION Essay Questions and Examiners’ Analyses and Performance Test Pennsylvania Board of Law Examiners

fact that any action taken by a federal court in the absence of jurisdiction is necessarily void. (citation

omitted) Thus, the removal statute is to be strictly construed and all doubts resolved in favor of remand.

If there is any doubt as to the propriety of removal, that case should not be removed to federal court.

(citation omitted)

Motions to remand, in turn, are governed by 28 U.S.C. § 1447. That statute provides, in relevant

part:

(c) A motion to remand the case on the basis of any defect other than lack of subject matter jurisdiction must be made within 30 days after the filing of the notice of removal under section 1446(a). If at any time before final judgment it appears that the district court lacks subject matter jurisdiction, the case shall be remanded. . . .

It has long been recognized that on a motion to remand, the removing party, as the party urging

the existence of jurisdiction, bears the burden of proving that jurisdiction exists. (citation omitted)

A district court must consider a number of settled precepts in ruling on a petition to remand a

case to state court for lack of diversity jurisdiction. Indeed, diversity jurisdiction is included in the

definition of original jurisdiction outlined in 28 U.S.C. § 1332 as follows:

(a) The district courts shall have original jurisdiction of all civil actions where the matter in controversy exceeds the sum or value of $75,000, exclusive of interest and costs, and is between [-](1) citizens of different States;

*** When a non-diverse party has been joined as a defendant, then in the absence of a substantial

federal question the removing defendant may avoid remand only by demonstrating that the non-diverse

party was fraudulently joined and he carries a heavy burden of persuasion in making this showing.

(citation omitted) Joinder is fraudulent "where there is no reasonable basis in fact or colorable ground

supporting the claim against the joined defendant, or no real intention in good faith to prosecute the

action against the defendants or seek a joint judgment." (citation omitted) But if there is even a

possibility that a state court would find that the complaint states a cause of action against any one of the

resident defendants, the federal court must find that joinder was proper and remand the case to state

court. (citation omitted) Furthermore, where there are colorable claims or defenses asserted against or

by diverse and non-diverse defendants alike, the court may not find that the non-diverse parties were

fraudulently joined based on its view of the merits of those claims or defenses. (citation omitted)

In this case, plaintiff alleges that Milnazik (acting as the agent, servant, workman and employee

of MONY) negotiated and sold the disability insurance policy at issue to her after conducting a personal

70

Page 75: JULY 2016 PENNSYLVANIA BAR EXAMINATION · JULY 2016 PENNSYLVANIA BAR EXAMINATION Essay Questions and Examiners’ Analyses and Performance Test Pennsylvania Board of Law Examiners

history interview and reviewing the application and other documents with her. (citation omitted) The

complaint further avers that she gave notice to the defendants of her disability and claim and submitted

proof of loss to them but that defendants refused to pay her the benefits to which she is entitled under the

policy. (citation omitted) Milnazik also allegedly failed to advise plaintiff that MONY would

"frivolously and capriciously deny sound medical evidence of total Disability, would not honor its

obligations under the policy without any sound basis in order to deny benefits by conducting a

perfunctory review of [Warner's] claim and a priori denying it without reasonable cause." (citation

omitted) All of these actions, plaintiff alleges, operated to breach defendants' "contractual undertakings"

with her. (citation omitted)

Against this background of alleged facts, Defendant argues that the joinder of the agency

defendant is fraudulent due to the fact that the policy was issued by MONY alone and therefore no

privity of contract exists between the agency defendant and plaintiff. (footnote omitted) While privity

may not ultimately be shown to exist, we cannot definitively find based only upon the materials now

before us that there is no reasonable basis in fact or colorable ground supporting the claim against

Milnazik, or no real intention in good faith to prosecute the action against the agency defendant or seek

a joint judgment. (citation omitted) We thus cannot find that defendant has met its burden of showing

that the sole reason for plaintiff's inclusion of Milnazik as a defendant in this suit was fraudulent and to

defeat diversity jurisdiction. (citation omitted) Accordingly, we are compelled to grant plaintiff's

motion(s) for remand.

* * *

71

Page 76: JULY 2016 PENNSYLVANIA BAR EXAMINATION · JULY 2016 PENNSYLVANIA BAR EXAMINATION Essay Questions and Examiners’ Analyses and Performance Test Pennsylvania Board of Law Examiners

Jerry ATCOVITZ and Roslyn Atcovitz, H/W, v. GULPH MILLS TENNIS CLUB, et al.

571 Pa. 580, 812 A.2d 1218 (2002)

Pennsylvania Supreme Court

We granted allowance of appeal in this case to determine whether a tennis club owes a duty of

care to its members to acquire and maintain an automated external defibrillator, hereinafter "AED," on

its premises for emergency use. (footnote omitted) For the reasons that follow, we hold that such clubs

do not owe a duty to have an AED available on their premises.

On January 16, 1996, Jerry Atcovitz suffered a stroke, secondary to a heart attack, while playing

tennis at the Gulph Mills Tennis Club. (footnote omitted) . . . Although he survived the incident,

Atcovitz "sustained severe and permanent injuries, including anoxic encephalopathy with multiple

permanent central nervous system disorders . . . .” (citation omitted)

***

Appellees, Jerry Atcovitz and his wife, Roslyn, sued Gulph Mills for negligence in the Court of

Common Pleas of Philadelphia County. (footnote omitted) Specifically, they claimed that, "had [Gulph

Mills] possessed an AED device and used it on [Atcovitz] promptly, his injuries would have been

significantly less and, therefore, that [Gulph Mills] is liable to him for damages." (citation omitted) In its

defense, Gulph Mills asserted that, at the time of Atcovitz's injury, its employees would not have been

permitted by law to use an AED.

In an attempt to preclude Gulph Mills from asserting its defense, Appellees moved for partial

summary judgment, which the trial court denied. Immediately prior to trial, however, Appellees orally

moved for reconsideration of their motion. At the same time, Gulph Mills cross-moved for summary

judgment. (footnote omitted) The trial court granted Gulph Mills's cross-motion for summary judgment

and dismissed the case. The court based its grant of summary judgment on the Emergency Medical

Services Act (footnote omitted), hereinafter the "EMS Act," and the regulations issued pursuant thereto.

The court concluded that, at the time of Atcovitz's injury, Gulph Mills's employees were legally

prohibited from using an AED. Thus, the court held that Gulph Mills "cannot be held negligent for

failure to use the device." (citation omitted)

Appellees filed a timely appeal to the Superior Court, which reversed the trial court's order

granting summary judgment. (citation omitted) The court opined that the trial court's reliance on the

EMS Act was inappropriate because it was designed for and aimed at the administration of emergency

services by trained and licensed professionals. As the EMS Act did not contain any provision

addressing emergency actions by untrained lay persons, i.e., Gulph Mills's employees, the court

72

Page 77: JULY 2016 PENNSYLVANIA BAR EXAMINATION · JULY 2016 PENNSYLVANIA BAR EXAMINATION Essay Questions and Examiners’ Analyses and Performance Test Pennsylvania Board of Law Examiners

concluded that the trial court's grant of summary judgment could not be supported by reference to the

EMS Act or its implementing regulations.

The court also addressed the effect of 42 Pa.C.S. § 8331.2, hereinafter the "AED Good

Samaritan Act," which provides "Good Samaritan civil immunity" for use of an AED in certain

instances. . . . Although the AED Good Samaritan Act was enacted after Atcovitz's injuries, the court

found that its passage evinced the Legislature's desire that use of AEDs not be restricted solely to trained

professionals. Accordingly, the court held that the trial court erred as a matter of law in granting Gulph

Mills's motion for summary judgment. (citation omitted) Subsequently, Gulph Mills petitioned this

Court for allowance of appeal, which we granted. (citation omitted)

***

The elements necessary to plead an action in negligence are: (1) the existence of a duty or

obligation recognized by law, requiring the actor to conform to a certain standard of conduct; (2) a

failure on the part of the defendant to conform to that duty, or a breach thereof; (3) a causal connection

between the defendant's breach and the resulting injury; and (4) actual loss or damage suffered by the

complainant. (citations omitted) Here, we must focus our analysis on the threshold element of duty.

(footnote omitted) Only therein may we resolve the fundamental question of whether the plaintiff's

interests are entitled to legal protection against the defendant's conduct.

"A duty, in negligence cases, may be defined as an obligation, to which the law will give

recognition and effect, to conform to a particular standard of conduct toward another." (citation omitted)

***

Within this construct, we must resolve whether Gulph Mills owed a duty to Atcovitz to acquire

and maintain an AED.

***

Neither the EMS Act nor the AED Good Samaritan Act imposed a duty upon Gulph Mills to

acquire, maintain, and use an AED. Appellees do not cite any other case, statute, or regulation that

would have imposed such a duty on Gulph Mills at the time of Atcovitz's injuries in January 1996.

Because Gulph Mills did not owe a duty to carry an AED, Appellees could not have established a prima

facie claim of negligence. (citation omitted) Thus, there was no genuine issue of material fact and

Gulph Mills was entitled to judgment as a matter of law.

***

73

Page 78: JULY 2016 PENNSYLVANIA BAR EXAMINATION · JULY 2016 PENNSYLVANIA BAR EXAMINATION Essay Questions and Examiners’ Analyses and Performance Test Pennsylvania Board of Law Examiners

Instructions

The performance test is designed to test an applicant’s ability to perform the

legal task that has been assigned using the factual information contained in the File and legal principles that are provided in the Library.

The File contains the only factual information that you should consider in performing the assigned task. The task to be completed is set forth in the first document in the File in the form of a memorandum to the applicant. The Library contains the only legal principles that you should consider to complete the assigned task. Although your general knowledge of the law may provide some background for analyzing the problem, the factual information contained in the File and the legal principles contained in the Library are the only materials that you should use in formulating your answer to the assigned task.

Your response should be written in the gray answer book or typed in answer screen number 3 of SofTest. Be sure to allow sufficient time for reading the materials, organizing your answer and completing the task assigned. Your answer should demonstrate an understanding of the relevant facts, recognition of the issues and the applicable principles of law and the reasoning that supports your answer. Your grade will be based on the content of your response and your ability to follow instructions in performing the assigned task.

The events depicted and the persons portrayed by the information in the File are fictitious and such information does not depict nor is it intended to depict or portray any actual person, company, or occurrence. Any similarity to any person, living or dead, or any occurrence is purely coincidental.

74

Page 79: JULY 2016 PENNSYLVANIA BAR EXAMINATION · JULY 2016 PENNSYLVANIA BAR EXAMINATION Essay Questions and Examiners’ Analyses and Performance Test Pennsylvania Board of Law Examiners

Question No. PT: Examiner’s Analysis

The applicant is assigned to draft a motion to remand the Plaintiff’s state-brought claim against

defendants from the United States District Court of the Northern District of Pennsylvania to the North

County Court of Common Pleas.

The applicant must utilize the provided statutes, rules, cases, and pleadings to support the motion for

remand, which should be based upon the untimeliness of the removal, as well as a lack of diversity,

thereby making removal improper.

Format 2 Points

Following directions is an important skill of every lawyer. The applicant is expected to follow the

directions provided concerning the format of the motion and its contents (i.e. – place a title at the top;

use numbered paragraphs for each material fact/legal assertion; address each argument in a separate

section of the motion; include informal citations to the authority and documents relied upon; and include

a request for relief). Formal Bluebook citations are not required.

The Removal Notice Was Not Timely Filed 4 Points

An action can be commenced by filing with the prothonotary a complaint or a writ of summons.

Pa.R.C.P. 1007.

A complaint is a pleading. Pa.R.C.P. 1017.

The Plaintiff, Peter Pratt, filed a complaint in this action with the prothonotary in the Court of Common

Pleas of North County, Pennsylvania, June 1, 2016. Complaint; Notice of Removal, ¶ 1.

The notice of removal of a civil action or proceeding shall be filed within 30 days after the receipt by the

defendant, through service or otherwise, of a copy of the initial pleading setting forth the claim for relief

upon which such action or proceeding is based. 28 U.S.C. § 1446(b)(1).

Defendants were served with the plaintiff’s complaint on June 6, 2016. Notice of Removal, ¶ 2.

Defendants filed their notice of removal on July 10, 2016, based upon the court’s diversity jurisdiction

under 28 U.S.C. § 1332. Notice of Removal.

Defendants had until July 6, 2016 to make a timely removal of plaintiff’s action, as July 6, 2016, was

thirty days from June 6, 2016, the time allowed by 28 U.S.C. § 1446(b)(1) to remove the matter.

Because Defendants waited longer than thirty days from the time they were served with the Complaint

before filing their Notice of Removal, removal was not timely, and the case should be remanded.

75

Page 80: JULY 2016 PENNSYLVANIA BAR EXAMINATION · JULY 2016 PENNSYLVANIA BAR EXAMINATION Essay Questions and Examiners’ Analyses and Performance Test Pennsylvania Board of Law Examiners

Removal Was Improper as the Court lacked Original Jurisdiction Because Complete Diversity

Does Not Exist

a. No original jurisdiction – lack of complete diversity. 4 Points

28 U.S.C. § 1441(a) authorizes the removal of civil actions from state court to federal court when the

action initiated in state court is one that could have been brought, originally, in a federal district court.

28 U.S.C. § 1441(a); Lincoln Property Co. v. Roche.

Defendants removed this matter based solely on diversity under 28 U.S.C. § 1332(a). Notice of

Removal.

Section 1332(a) provides inter alia, that the district courts shall have original jurisdiction of all civil

actions where the matter in controversy exceeds the sum or value of $75,000, exclusive of interest and

costs, and is between citizens of different states. 28 U.S.C. § 1332.

The United States Supreme Court has read the statutory formulation “between … citizens of different

States” to require complete diversity between all plaintiffs and all defendants . . . .” Lincoln Property

Co. v. Roche.

Plaintiff is a citizen of Pennsylvania. Complaint ¶ 1.

Defendant Dunn is a citizen of Pennsylvania. Complaint ¶ 3.

As the Plaintiff and Defendant Dunn are citizens of Pennsylvania “complete diversity between all

plaintiffs and all defendants” does not exist. Complaint; Lincoln Property Co. v. Roche.

Applicants should conclude that the absence of complete diversity between Plaintiff and Defendant

Dunn deprives the federal district court of original jurisdiction; thus, the matter should be remanded.

b. When removal is based on diversity of citizenship, defendant cannot be a citizen of

forum state. 4 Points

When federal-court jurisdiction is predicated on the parties’ diversity of citizenship, removal is

permissible “only if none of the parties in interest properly joined and served as defendants is a citizen

of the State in which [the] action [was] brought.” 28 U.S.C. §1441(b); Lincoln Property Co. v. Roche.

Peter Pratt brought this cause of action in the Court of Common Pleas of North County, Pennsylvania,

making Pennsylvania the forum state. Complaint; Notice of Removal.

Defendant Deborah Dunn is a citizen of Pennsylvania. Complaint, ¶ 3.

Thus, Deborah Dunn is a defendant and a citizen of the forum state. Complaint, ¶ 3.

As Dunn is a citizen of Pennsylvania, which is the forum state, the cause of action was improperly

removed and should be remanded to the Court of Common Pleas of North County, Pennsylvania.

c. Inclusion of Dunn as Defendant Not Fraudulent 6 Points

76

Page 81: JULY 2016 PENNSYLVANIA BAR EXAMINATION · JULY 2016 PENNSYLVANIA BAR EXAMINATION Essay Questions and Examiners’ Analyses and Performance Test Pennsylvania Board of Law Examiners

Credit will be given to applicants who discuss fraudulent joinder in conjunction with a complete

diversity argument or a forum defendant argument, as it is an appropriate discussion for both.

In the notice of removal, defendants have alleged that Defendant Dunn was improperly joined as a party

to defeat diversity. Notice of removal ¶¶ 7, 8.

When a non-diverse party has been joined as a defendant, then in the absence of a substantial federal

question the removing defendant may avoid remand only by demonstrating that the non-diverse party

was fraudulently joined and he carries a heavy burden of persuasion in making this showing. Warner v.

Mutual Life.

Joinder is fraudulent "where there is no reasonable basis in fact or colorable ground supporting the claim

against the joined defendant, or no real intention in good faith to prosecute the action against the

defendants or seek a joint judgment." Warner v. Mutual Life.

If there is even a possibility that a state court would find that the complaint states a cause of action

against any one of the resident defendants, the federal court must find that joinder was proper and

remand the case to state court. Warner v. Mutual Life.

Where there are colorable claims or defenses asserted against or by diverse and non-diverse defendants

alike, the court may not find that the non-diverse parties were fraudulently joined based on its view of

the merits of those claims or defenses. Warner v. Mutual Life.

Plaintiff has asserted a cause of action against Deborah Dunn for negligence in the mishandling of his

insurance claim. Complaint ¶¶ 15-21.

The elements necessary to plead an action in negligence are: (1) the existence of a duty or obligation

recognized by law, requiring the actor to conform to a certain standard of conduct; (2) a failure on the

part of the defendant to conform to that duty, or a breach thereof; (3) a causal connection between the

defendant's breach and the resulting injury; and (4) actual loss or damage suffered by the complainant.

Atcovitz v. Gulph Mills Tennis Club.

Pratt had an insurance policy purchased from First Insurance Co. Complaint ¶ 5.

First Insurance employed Deborah Dunn and assigned her to handle Pratt’s insurance claim. Complaint

¶¶ 3, 8.

As stated in the complaint, Deborah Dunn had a duty to properly handle plaintiff’s claim pursuant to

Pennsylvania law. Complaint, ¶ 15.

Dunn cited to other individual’s medical records instead of Pratt’s medical records in denying his claim.

Complaint ¶ 17.

Dunn failed and refused to speak to Pratt about his insurance claim. Complaint ¶ 18.

Dunn failed to direct First to pay plaintiff’s claim and mishandled his claim at every step of the way.

Complaint ¶ 18.

77

Page 82: JULY 2016 PENNSYLVANIA BAR EXAMINATION · JULY 2016 PENNSYLVANIA BAR EXAMINATION Essay Questions and Examiners’ Analyses and Performance Test Pennsylvania Board of Law Examiners

Plaintiff’s complaint states that Dunn’s negligence was the direct and proximate cause of plaintiff’s

harm, the lack of payment of the $100,000 underinsured motorists claim and resulting financial harm.

Complaint, ¶ 19-20.

Finally, plaintiff’s complaint states that Dunn’s failure resulted in actual financial harm to plaintiff.

Complaint, ¶ 20.

Plaintiff has pled in his complaint all elements of a claim for negligence against defendant Dunn.

Complaint and Atcovitz v. Gulph Mills Tennis Club.

Thus, plaintiff has stated a colorable claim of negligence against the non-diverse defendant, making it

possible that a state court would find that the complaint states a cause of action; and the federal court

must find that the joinder was proper and remand the case to state court.

The defendants will be unable to meet the heavy burden of persuasion in establishing that the non-

diverse party was fraudulently joined.

Therefore, the federal court may not find that plaintiff has fraudulently joined the non-diverse defendant.

Because Deborah Dunn is a properly joined Pennsylvania citizen, and the plaintiff is a Pennsylvania

citizen, there is an absence of complete diversity. Additionally, because Dunn is a citizen of the forum

state removal is improper. 28 U.S.C.§1441(b)(2) and 28 U.S.C. § 1332.

78